关键词不能为空

当前您在: 主页 > 英语 >

【附20套高考模拟试题】贵州省2020年7月普通高中学业水平考试英语试卷含答案

作者:高考题库网
来源:https://www.bjmy2z.cn/gaokao
2020-11-02 16:22
tags:七月英文

ultimate是什么意思中文翻译-博斯普鲁斯海峡

2020年11月2日发(作者:严复)


贵州省2020年7月普通高中学业水平考试英语试卷

第一部分 (共20小题每,小题1.5分,满分30分)
1

The little boy stared at the strange man questioningly, not ________ whether to believe what he had said.
A

to know
C.known D.having known
B

knowing
2

—Would you please wait for a moment while I telephone the reception desk to check?
—________.
A

I wouldn't mind that
C.I mean it D.You wish
3

The 114 colorful clay Warriors ____ at No. 1 pit, ______ in height from 1.8m to 2m, have black hair,
green, white or pink faces, and black or brown eyes.
A

unearthed; ranging B

unearthing; ranging
C.unearthed; ranged D.are unearthed; are ranging
4

Looking people in the eye ______ sometimes make them nervous and embarrassed.
A

must
C.should D.might
B

can
B

Sound like fun
5

_______ more energy to my study instead of being crazy about computer games, I would be sitting in a
comfortable office now.
A

If I devoted
C.Would I be devoted
B

Had I devoted
D.Should I be devoted
6

—A study suggests reducing energy demand in the future may ________ urban areas.
—That’s true. Cities need more energy than small towns or other rural areas.
A

catch on B

act on
C.hang on D.center on
7

Mother told me in a phone call that she had written to me a week before, but I haven’t
received__________.
A.one B.it C.something D.anything
8
.-
The online shopkeeper has made an apology for his rude behavior.

OK. If you’re still not satisfied, you can _______ compensation.
A

claim
C.sacrifice D.dismiss
9

—I'm sorry. I shouldn't have been so rude to you.
—You ________ something not very nice to me, but that's OK.
A.have said B.had said C.were saying D.did say
10

Never before ________ the famous museum was just a stone's thro away from their school, so out
________.
B

afford


A

had they known; went all they B

they had known; went all they
C.had they known; they all went D.they had known; they all went
11

She must have ranked her birthday presents in order of _because the top one is her favorite


A.value B.time C.interest D.preference
12

He was admitted to Harvard University, ________ was just what his parents expected.
A

who
C.what D.where
13

Guangdong province rolled out new guidelines, _____ offensive nicknames and online violence as school
bullying.
A

defining
C.to define D.having defined
14

____regular training in nursing, she could hardly cope with the work at first.
A

Not received B

Since receiving
C.Having not received D.Not having received
15

Some business owners are keen on public welfare. This is local services have been funded.
A.whether B.what C.where D.how
B

defined
B

which
16

We are living in an age most of things are done on cell phones.
A

which
C.whose D.when
B

that
17

---Excuse me, could you tell me the time, please?
---Sorry, I don’t have a watch with me.
--- ___________
A

Thanks a lot. B

What a pity.
C.I’m sorry to hear that D.Thanks all the same
18

Why are some brands more popular than others ______ the products are of similar quality?
A.as though B.even if C.so that D.in case
19

_____ working ability

you are no worse than the others

it is just you who don’t put your heart in your
work


A.In favor of B.In honor of C.In terms of D.In case of
20

There’s another way to survive this competition -- a way no one ever seems to tell you about, _____ you
have to learn for yourself.
A.one B.it C.that D.another
第二部分 阅读理解(满分40分)阅读下列短文,从每题所给的A、B、C、D四个选项中,选出最佳选
项。
21
.(
6
分)
Don't get mad the next time you catch your teenager texting when he promised to be
studying. He simply may not be able to resist. A University of lowa(UI) study found teenagers are far more
sensitive than adults to the immediate effect or reward of their behaviors. The findings may help explain
why the initial rush of texting may be more attractive for adolescents than the long-term pay off of


studying.
Vaidya

an author of the study.

no longer in a teenager's best interest to continue, they will, because the effect of the reward is still there
and lasts much longer in adolescents than in adults .
For parents,that means limiting distraction (
分心的事情)
so teenagers can make better choices. Take
the homework and social media dilemma: At 9 p.m., shut off everything except a computer that has no
access to Facehook or Twitter, the researchers advise.
technology, so they can
develop those impulse-control skills.”
In their study,Vaidya and co-author Shaun Vecera note researchers generally believe teenagers are
impulsive
(冲动的
)

make bad decisions

and engage in risky behavior because the frontal lobes
(额叶)
of their trains are not fully developed. But the UI researchers wondered. whether something more
fundamental was going on with adolescents to cause behaviors independent of higher-level reasoning.

adulthood,
decision-making. “We've been trying to understand the reward process in adolescence and whether
there is more to adolescence behavior than an under- developed frontal lobe,”he their
study ,the researchers persuaded 40 adolescents, aged 13 and 16,and 40 adults, aged 20 and 35 to
participate.
In the future,researchers hope to look into the psychological and neurological
(神经学上的)
aspects of
their results.
1

What does the passage mainly tell us?
A

The initial rush of texting is less attractive for adolescents than the long-term pay off of studying.
B

Always, rewards are attractive to teenagers.
C

Resistance can be controlled well by adolescents.
D

Getting rewards is the greatest motivation for adolescents to study.
2

Which statement agrees with Vaidya's idea?
A

The influence of the reward is weak in adolescents.
B

Parents should help children in making decisions.
C

Children should have access to the Internet.
D

Children need help in refocusing their attention.
3

What result does teenagers' brain underdevelopment lead to?


A

Doing things after some thought.
B

Making good decisions.
C

Joining in dangerous actions.
D

Escaping risky behavior.
4

How did the researchers carry out their study?
A

By making a comparison of brain examinations.
B

By examining adults’ brain.
C

By examining teenage brain.
D

By building the train’s reward system.
22
.(
8
分)
Martha Graham, born in 1894, was one of the most famous dancers and creators of dance,
whose influence on dance has been compared with the influence Picasso had on the modern visual arts,
Stravinsky had on music, or Frank Lloyd Wright had on architecture. She created almost two hundred
dance pieces. She is often called the Mother of Modern Dance.
Earlier in her life, however, Martha did not know that she would become a dancer. At that time, the
dancers were looked down on. So Martha's parents didn't approve of her desire to dance at the beginning.
Until 1916, she began her studies at the newly created Denishawn School of Dancing and Related Arts,
founded by Ruth St. Denis and Ted Shawn, where Graham worked very hard to improve her ability to
dance because she was considered too old to begin dancing.
In 1936, Graham made her defining work, beginning of a new era in
modern dance. The dance brought serious issues to the stage for the general public in a dramatic manner.
Influenced by the Wall Street Crash of 1929, the Great Depression that followed, and the Spanish Civil War,
it focused on depression and isolation, reflected in the dark nature of both the set and costumes.
(服装)

Graham continued to dance past the age of seventy. Once again, she was met with criticism from people
who came to watch her shows. But she didn't give up. Her last completed ballet was 1990's Maple Leaf Rag.

group shows some of their methods.
Martha Graham received many awards during her lifetime, including the Presidential Medal of
Freedom in nineteen seventy-six. She was the first dancer to receive the country's highest civilian honor.
She died in 1991 at the age of ninety-six. In 1998, Time magazine listed her as the
and as one of the most important people of the twentieth century.
1

Some famous artists mentioned in Paragraph 1 show Martha Graham________ .
A

was influenced greatly by them
B

kept in touch with them regularly


C

had a great effect on modern dance
D

liked to make more friends with them
2

Why didn't Martha's parents approve of her desire to dance at first?
A

She was too old to learn American dance.
B

American dance was seen as a lower art form.
C

There was no academic school to teach dancing.
D

Her family was too poor to afford her tuition.
3

When Martha Graham continued to dance in her 70s, she_________.
A

was warmly welcomed by the youth
B

achieved huge success once again[
C

received many awards for her age
D

was attacked by people
4

The text is mainly about___________.
A

the background of modern dance in America
B

the development of modern dance in America
C

Martha Graham and her achievements in dance
D.the awards Martha Graham won in her life
23
.(
8
分)
Years ago, students applying to colleges and universities had to have their applications and
personal essays printed out and then “snail-mailed”. Flash forward to today’s world of Facebook,
Instagram and WeChat, where photos, videos and mobile phones rule. Fortunately, several new tools are
making it easier for college applicants to use technology to show off their personalities, skills and creativity.
ZEEMEE
One of the more popular innovations in college applications is ZeeMee, a free mobile app that allows
students to upload personal profiles and videos to create visual resumes. More than 220 colleges and
universities offer a ZeeMee option as part of the application process. The video can show the unique
creative aspects of the applicants.
THE COALITION LOCKER
Another relatively new tool comes from the Coalition for Access, Affordability and Success, now used
by more than 110 member colleges and universities. Among the group’s stated goals are getting students
engaged in college prep early and using technology to create a supportive application process that
encourages reflection and self- discovery.
Students who sign up for the free platform get access to a digital “locker”, which they can use all
through high school to save class papers, artwork, newspaper stories, videos and photos to share with
colleges when they eventually apply.


VIDEO — CHAT INTERVIEWS
At some schools, an interview with an admission officer is an important part of the application process.
To make it easier, several virtual interviewing tools have popped up, such as a video-based platform from
Kira. More than 140 universities worldwide use Kira’s video assessment tool to conduct real-time
interviews with prospective students. During the interview, candidates are shown the questions prerecorded
by the particular school and must respond in real time by talking into the camera on their laptop or phone.
In addition to showing their ability, to speak fluently in English and think on their feet, it helps admission
officers determine the “motivation, commitment and drive” of applicants.
UNIQUE SCHOOL TOOLS
Some universities have come up with their own ways to allow students to show their skills and
personalities. For example, the Massachusetts Institute of Technology, encourages applicants to display
their technical abilities and creativity by submitting a “Maker Portfolio” —a written or videotaped
description of a project that shows creative and problem-solving skills. While a new app would fit the bill,
the project could be anything from an origami design to a potato cannon.
1

Which of the apps give students more guidance from their high school?
A

ZEEMEE. B

THE COALITION LOCKER.
C

VIDEO — CHAT INTERVIEWS. D

UNIQUE SCHOOL TOOLS.
2

What does the underlined word “it” in the fourth part of the passage refer to?
A

The video interview. B

The description.
C

The application. D

The student’s ability.
3

What is the common feature required by colleges and universities according to the passage?
A

Creativity. B

Braveness.
C

Willingness to share. D

Technical ability.
24

Doctor Seuss was born in 1904. By the middle 1950s, he had become one of the best loved and

8
分)
most successful children's book writers in the world. His books are very popular with young readers. They
enjoy the invented words and the pictures of unusual funny animals and plants.
In 1954, life magazine published(
刊登
) a report about school children who could not read. The report
said many children's books were not interesting. Doctor Seuss strongly hoped to help children and decided
to write books that were interesting and easy to read. He used words with the same ending sound, like fish
and wish. He did not receive training in art. Yet, he drew the pictures for most of his books.
In 1957, Dr. Seuss wrote The Cat in the Hat. He used less than 230 words to write the book and even a
six- year-old should be able to read it. It was a fun story and easy to read. Children loved it. Their parents
loved it, too. Today it is still one of the stories they like best. The success of The Cat in the Hat made him


want to write more books for children. In 1960, he wrote a book using less than fifty words. The book is
called Green Eggs and Ham.
In 1984, Doctor Seuss won a Pulitzer Prize (
普利策奖
). He was honored for the education and enjoyment
his books provided American children and their parents.
He died at the age of 87, but his influence remains. Millions of his books have been sold worldwide.
People say his books helped change the way American children learned to read. Yet, his books are loved by
people of all ages. Doctor Seuss once said

“I do not write for children. I write for people.”
1

Doctor Seuss learned from the magazine that _________.
A

some school children could not read with interest.
B

many children's books were interesting
C

children wanted to learn to read
D

a writer for children was wanted
2

People like his books because the books ________.
A

are cheap and easy to get B

were written in different languages
C

are easy and interesting to read D

were written with invented words
3

He wrote the book The Cat in the Hat at the age of ________.
A

50 B

53
C

56 D

87
4

Which of the following is TRUE according to the passage?
A

Doctor Seuss wrote Green Eggs and Ham with over 230 words.
B

Doctor Seuss wrote books only for children in the United States.
C

The Cat in the Hat was written only for six-year-old children.
D

His books provided education and enjoyment for Americans.
25
.(
10
分)
Every living thing on Earth is either a plant or an animal, right? Wrong! Many organisms
(
有机体
) in our world are neither. Some of them float through the air, and some lie hidden in the ground.
They do not need sunlight to survive, and if the weather becomes too cold, they can become inactive until
conditions improve.
What are these? They are called fungi (
菌类
), and you see them almost every day. Many kinds of fungi
seem disgusting, such as the green black things that appear on the food which are left too long in the
refrigerator. But other kinds are not so bad. For example, the yeast (
酵母
) that is used to make bread and
mushrooms are both fungi.
What makes fungi different from plants and animals? An animal can move around by itself. Fungi get
around too, but they don’t have feet, wings, or tails. They have to wait for wind or some other outside force


to move them. Plants are different from fungi because they have chlorophyll (
叶绿素
). It helps them make
food from the energy in sunlight. Fungi do not have chlorophyll.
Some fungi are bad, while others are not. Some fungi cause diseases, such as athlete’s foot, which
makes feet uncomfortable. Other kinds of fungi can be used to make medicines, which have saved many
lives.
Fungi clean up more than wounds, though. They are the world’s first recyclers. Without fungi, our
world would be a mess. Since fungi cannot make their own food as plants do, they must get their food
elsewhere. Many fungi get their food from dead plants. They break them down and turn them into soil. The
fungi get a meal, and the world gets a housecleaning.
1

What does the underlined word “disgusting” most probably mean?
A

strong. B

unpleasant.
C

active. D

small.
2

According to the passage, .
A

fungi can move everywhere by themselves
B

fungi are unable to survive in cold conditions
C

some fungi can make food from the energy in sunlight
D

some fungi are useful to humans and the environment
3

Where can you most probably find this passage?
A

In a social report.
B

In a biology book.
C

In a geography magazine.
D

In a farming book.
第三部分 语言知识运用(共两节)第一节(每小题1.5分,满分30分)阅读下面短文 ,从短文后各题所
给的A、B、C和D四个选项中,选出可以填入空白处的最佳选项.
26

Born in America, I spoke English ,not Chinese , the language of my ancestors . When I

30
分)
was three, my parents flashed cards with Chinese 1 at my face , but I pushed them 2 . My
mom believed I would learn 3 I was ready. But the 4 never came.
On a Chinese New Year’s Eve, my uncle spoke to me in Chinese, but all I could do was 5 at him ,
confused , scratching my head . “Still can’t speak Chinese? ” He 6 me , “You can’t even buy a fish in
Chinatown .”
“Hey, this is America , not China. I’ll get some 7 with or without Chinese.” I replied and turned
to my mom for 8 . “Remember to ask for fresh fish, Xin Xian Yu,” she said ,handing over a $$20 bill .
I 9 the words, running downstairs into the streets of Chinatown.
I found the fish 10 surrounded in a sea of customers. “I’d like to buy some fresh fish,” I should


to the fish man. But he 11 my English words and turned to serve the next customer .The laugh of the
people behind increased 12 their impatience. With every 13 , the breath of the dragons (

)
on my back grew stronger—my blood boiling— 14 me to cry out . “ Xian Sheng Yu , please.” “Very
Xian Sheng,” I repeated .The crowd erupted into laughter . My face turned 15 and I ran back
home 16 , except for the $$20 bill I held tightly in my pocket.
Should I laugh or cry? They’re Chinese. I should feel right at 17 . Instead , I was the joke , a
disgrace (
丢脸
)to the language.
Sometimes, I laugh at my fish 18 , but , in the end, the joke is on 19 . Every laugh is a
culture 20 every laugh is my heritage (
传统
) fading away.
1

A

custom
2

A

ahead
3

A

when
4

A

success
5

A

aiming
6

A

cared about
7

A

right now
8

A

decision
9

A

repeated
10

A

farm
11

A

guessed
12

A

by
13

A

second
14

A

forcing
15

A

bright
B

games
B

around
B

before
B

study
B

joking
B

laughed at
B

from now
B

permission
B

reviewed
B

stand
B

forget
B

as
B

effort
B

allowing
B

blank
C

characters
C

along
C

unless
C

time
C

nodding
C

argued with
C

at times
C

information
C

spelled
C

pond
C

doubted
C

with
C

desire
C

persuading
C

pale
B

tongue-tied
C

risk
C

challenge
C

me
C

divided
D

language
D

aside
D

until
D

attempt
D

staring
D

asked after
D

in time
D

preparation
D

kept
D

market
D

ignored
D

from
D

movement
D

leading
D

red
C

empty-handed D

broken- hearted
D

root
D

incident
D

them
D

reflected
16

A

open-mouthed
17

A

service
18

A

trade
19

A

it
20

A

thrown
B

home
B

deed
B

us
B

lost
第二节(每小题1.5分,满分15分)阅读下面材料,在空 白处填入1个适当的单词或括号内单词的正确
形式。
27
.(
15
分)
What is clone? The noun “clone” and the verb “to clone” are not used consistently. In
biology a clone is a cell or an organism, which is 1

(genetic) the same as another cell or organism. Many
simple organisms such as bacteria reproduce themselves by 2

(copy) their DNA and splitting (
分裂
) in half.


The verb “to clone” refers 3

the process of creating cloned cells or organisms. The process differs,
4

(depend) on the kinds of cells used in the cloning procedure and the desired result. Usually, 5

scientists
clone an animal, they take the nucleus (

) of a cell and place it into an egg cell from 6

the nucleus has
been removed. The egg cell then divides to produce 7

embryo that develops an animal if the procedures
work as 8

(plan). Different from other cloning cases, Dolly was created from a specialized adult cell,
9

from a very early embryonic cell in which no specialized 10

(tissue) begin to develop.
第四部分 写作(共两节)第一节 短文改错(满分10分)
28
.(
10
分)

短文改错

假 定英语课上老师要求同桌之间交换修改作文,请你修改你同桌写的以下作文。文中共有
10
处错 误,
每句中最多有两处。错误涉及一个单词的增加、删除或修改。

增加:在缺词处加一个漏词符号(
^
),并在其下面写出该加的词。

删除:把多余的词用斜线(

)划掉。

修改:在错的词下画一横线,并在该词下面写出修改后的词。

注意:
1
每处错误及其修改均仅限一词;

2
只允许修改< br>10
处,多者(从第
11
处起)不计分。

Last week I went to visit my former neighbor, Mr

Yang

He and I used to living next to each other for
many years

About half a year ago, news came which the old building, along with many other similar ones,
were going to be pulled down to make a room for a main street

So they had to move apart

Mr

Yang
now lives in the suburbs near a beautifully park

His apartment is much big than before

The only
problem is that it took quite a long time to get to the downtown area

Therefore, Mr

Yang doesn’t seem
to worry about the long distances

He says that a new subway line will be built in a few years and he is
sure life will be better in the future


第二节 书面表达(满分25分)
29
.(25
分)假设你是李华,校学生会为了提升学生的文化素养,举办了《朗读者》活动,请你写信邀请
你的交换生朋友
Jim
和你一起参加。

信的内容包括:

1.
你邀请他一起参加的原因。

2.
报名方式和截止时间。

注意:
1.
词数
80
左右;

2.
可适当增加细节,以使行文连贯。

________________ __________________________________________________ _________________________
_
_______________ __________________________________________________ __________________________
_
______________ __________________________________________________ ___________________________


_
______ __________________________________________________ ___________________________________
_
_____ __________________________________________________ ____________________________________
_
____ __________________________________________________ _____________________________________
_
___ __________________________________________________ ______________________
参考答案
第一部分 (共20小题每,小题1.5分,满分30分)
1.B
2.A
3.A
4.B
5.B
6.D
7.D
8.A
9.D
10.C
11.D
12.B
13.A
14.D
15.D
16.D
17.D
18.B
19.C
20.A

第二部分 阅读理解(满分40分)阅读下列短文,从每题所给的A、B、C、D四个选项中,选出最佳选
项。
21

1

B
2

D
3

C
4、 A
22

China has more than 30 intangible cultural heritage(
非物质文化遗产
) recognized by the UNESCO,
including paper-cutting, the Dragon Boat Festival, Peking Opera, acupuncture(
针灸
) and so on. The
organization adopted a decision that China’s “The Twenty-Four Solar Terms”(
二十四节气
) should be put
on the Representative List of the Intangible Cultural Heritage of Humanity in 2016 in Ethiopia.
The Twenty-Four Solar Terms, knowledge of time and practices in agriculture, starts from the


beginning of Spring and ends with the Greater Cold, moving in cycles. It developed through thee
observation of the sun’s annual movement in China. In ancient time, the method of tugui, earth sundial(


), was used to measure the shadow of the sun for determining the solar terms.
The Twenty-Four Solar Terms came into being and developed in close relationship with Chinese
agricultural production. At the initial
(最初的)
stage of agricultural development, people began to explore
the seasonal rules in the agricultural production to meet the needs in seeding, harvesting and other
activities. Gradually, they formed the concept of “seed in spring, grow in summer, harvest in autumn and
store in winter”. During the spring and autumn periods, the agricultural production was highly influenced
by the seasonal changes, thus forming the concept of Solar Terms.
As a traditional Chinese knowledge system of time with a history of thousands of years, the
Twenty- Four Solar Terms clearly expresses the concepts of respect for nature, and harmony between man
and nature. Created by Chinese ancestors, it has functioned as a complete set of weather calendar(
日历
) to
guide the agricultural production in China. It has also been introduced into North Korea, Japan and other
neighboring countries and still used in Japan. The Chinese heritage has provably influenced the people’s
way of thinking and behaving and will continue to be an important carrier of Chinese cultural identity.
5

According to the first two paragraphs, the Twenty-Four Solar Terms______.
A

was used to measure the shadow of the sun
B

has not been listed as one intangible cultural heritage of humanity
C

repeats from the Beginning of Spring to the Greater Cold every year
D

is the best intangible cultural heritage recognized by the UNESCO
6

We can learn from the text that _______ in China in the past.
A

the Twenty-Four Solar Terms only influenced spring and autumn
B

the Twenty-Four Solar Terms had something to do with agriculture
C

people cared about the changes of weather just for fun
D

agricultural production highly influenced the seasonal changes
7

We can infer from the text that the Twenty-Four Solar Terms _______.
A

is an agricultural calendar merely used in China
B

is strongly influenced by North Korea and Japan
C

is scarcely connected with natural rules
D.is part of traditional Chinese culture
23

1

B
2

C
3、A
24

1

A


2

C
3

B
4、D
25
.【推理关系】题干
What does the underlined word “disgusting” most probably mean??
文章内容
not bad

disgusting
是反义词
?B.
选项
unpleasant
【名师点睛】猜测词义题之依据反义词和对比关系猜测词义

依据反义词和对比关系猜测词义

运用对比手法描述事物或现象是文章写作的一种常 用的修辞手法。在这种修辞手段下,作者常借用一些信
号词来提供相反信息,从而表明一个词与前面的另 一个词互为反义。常用的信号词有:

butyethowevernevertheless; whilebut otherwiseor else; unlikeinstead rather than; by
contrastcompared to; on the contrary on the other hand
等。

1

D
细节理解题。根据第二段最后一句
“They do not need sunlight to survive, if the weather becomes too cold,
they can become inactive until conditions impr ove.”
可知
B
是错误的。根据第三段可知这些菌类也移动但要
靠风或其他 的外力,自己不能,故
A
是错误的。根据第三段说菌类靠叶绿素来获得能量,故
C是错误的。
根据最后两段可知有些菌类不仅可用于制造药物,它还能给世界清洁,因此答案
D
符合题意,故选
D


2

B
推理判 断题。本文主要讲述了地球上的真菌的种类,如何生存,对人类及环境的作用等知识。因此它属于
生物学 内容,有可能出现在生物书上,故选B。

第三部分 语言知识运用(共两节)第一节(每小 题1.5分,满分30分)阅读下面短文,从短文后各题所
给的A、B、C和D四个选项中,选出可以填 入空白处的最佳选项.
26

1

C
2

D
3

A
4

C
5

D
6

B
7

A
8

B
9

A
10

B
11

D
12

C
13

A


14

A
15

D
16

C
17

B
18

B
19

C
20、B

第二节(每小题1.5分,满分1 5分)阅读下面材料,在空白处填入1个适当的单词或括号内单词的正确
形式。
27

1

genetically
2

copying
3

to
4

depending
5

when
6

which
7

an
8

planned
9

not
10

tissues

第四部分 写作(共两节)第一节 短文改错(满分10分)
28

living ——live
which the——which that
were——was
make a room ——a
去掉

So they——So we
Beautifully——beautiful
Big—— bigger
Took——takes
Therefore——However
Distances——distance

第二节 书面表达(满分25分)
29

Dear Jim,
I am entering for the Readers organized by the Student Union this June. I am writing to invite you to


take part together.
As an exchange student, you are not only passionately fond of the Chinese culture but also have a good
knowledge of Chinese literature. Why not take the chance to challenge yourself? And with the company of
each other, we can work together and get better prepared.
If you are interested, don’t hesitate to sign up at the Student Union no later than May 30
th
.
Yours sincerely
Li Hua
高考模拟英语试卷
第Ⅰ卷(共95分)
第一节:单项填空 (共25题; 每小题1分,满分25分)
从A、B、C、D四个选项中,选出可以填入空白处的最佳选项,并在答题卡将该项涂黑。
1. Hiking is ______ great fun. You will get close to ______ nature and take exercise at the same time.
A. a; the B. a; / C. /; the D. /; /
2. —What are you mailing, Ryan?
—A textbook_____ a new method of teaching physics. I want my friends to take a look at it.
A is based on B based on C basing on D which based upon
3. If you want to do international trade successfully, ______ of English is _______.
A. good command; a must B. a good command; a need
C. a good command; a must D. good command; must
4. No product is allowed to go into the market if it fails to ______ the quality standard.
A. live B. come up to C. meet D. go through
5. There is ______ is called Mr. Smith in our workshop.
A. no such man as B. no such a man as C. no such man that D. no such a man that
6. —The book deserves ____ a second time.
—Sorry, I can’t follow you clearly.
—I said the article is worth ______ again.
A. studying; to study B. to be studied; to be studied
C. studying; studying D. studying; being studied
7. Tom admitted ______ in the examination and he was not admitted _____ the school at last.
A. to cheat; to B. cheating; to C. to cheat; as D. cheating; as
8. I never fancied Mary—so weak a girl—_______ the football match and did pretty well.
A. taking part in B. joining in C. to join in D. to take part in
9. —I never fancied Mary—so weak a girl— ______ the football match and did pretty well.


A. taking part B. joining in C. to join D. to take part in
10. —Someone______ have watered the garden this morning, didn’t he?
—Yes, James did. I _______ have done but I have been washing the whole morning.
A. would; would B. should; might C. could; must D. must; should
11. — You shouldn’t have gone there alone last night.
— But I ________, because iao Wang went there, too.
A. didn’t B. had to C. did D. should
12. Millions of calculations involved, had they been done by hand, _______ all practical value by the
time they were finished.
A. would have lost B. could lose C. ought to have lost D. might lose
13. It was ______ back home after the experiment.
A. not until midnight did he go
B. until midnight that he didn’t go
C. not until midnight that he went
D. until midnight when he didn’t go
14. I was so relieved that I didn’t have to do the after-dinner speech – I felt a weight __________ my mind!
A. taking down B. taken off C. was taken down D. to take from

15 — Are you still working in that computer company, Walt?
— ________. It’s three years since I worked there.
A. Yes, I am. B. No, I’m not.
16. — Why not go to the ball tonight?
—If you go, _______.
A. so will I B. so do I C. so I will D. so I do
C. No, seldom D. Yes, all the time.
17. Please ______a situation where you can put the daily expressions.
A. make out B. make up C. make up for D. put up
18. He insisted that he ________ in good health and _______ to work there.
A. was, be sent B. is, is sent C. be, was sent D. be, send
19. — Is it for two months ______ they have stayed here?
— No, it is only three weeks ______ they arrived here.
A. that; since B. that; when C. when; that D. since; before
20. ______your composition carefully, some spelling mistakes can be avoided.
A. Having checked B. Check C. If you check D. To check
21. “Don’t you think it necessary that he ____ to Miami but to New York?”


“I agree, but the problem is _____ he has refused to.”
A. will not be sent; that B. not be sent; that
C. should not nr sent; what D. should not send; that
22. Eco-travel is a form of travel ___ combines normal tourism with learning, _____ a way to find out what
can be done to help animals, plants and people.
A. what; as well as B. that, as well C. which; as well as D. which; as well
23. To go on business, I will have to stay ____ for some time, so I will ask someone to ___.
A. away; take my place B. off; instead of me
C. off; take the place of mine D. away; take place of me
24. I mean _____ abroad and nothing is going to stop me, though that means ______ my parents for a long
time.
A. going; leaving B. going; to leave C. to go; to leave D. to go; leaving
25. The young lady is ____ a beauty; she is ______ than pretty.
A. more than; more smart
C. not only; smarter

第二部分:完形填空(共20小题,每小题1分;满分20分)
阅读下面短文,掌握其大意,然后从26~45各题所给的四个选项中,选出一个最佳答案。

My friend Michelle is blind, but you’d never know it. She makes such good use of her other 26 ,
including her “sixth sense”, that she rarely gives the impression that she’s 27 anything.
Michelle looks after her children pretty much like the rest of us, 28 that she doesn’t push too hard
on them, 29 really benefit a lot from her relaxed attitude. She knows when to clean the house, and she
moves around so fast that 30 often don't realize she's blind.
I 31 this the first time after my six-year-old daughter, ayla, went to play there. When ayla came
home, she was very 32 about her day. She told me they had baked cookies, played games and done art
projects. But she was 33 excited about her finger-painting project.
“Mom, guess what?” said ayla, all smiles. “I learned how to 34 colors today! Blue and red make
purple, and yellow and blue make green! And Michelle 35 with us”.
To my great 36 , my child had learnt about color from a blind friend! Then ayla continued,
“Michelle told me my 37 showed joy, pride and a sense of accomplishment. She really 38 what I was
doing!” ayla said she had never known how good finger paints felt 39 Michelle showed her how to
paint without looking at her paper.

B. more than; smarter
D. no more than; more clever


I realized ayla didn’t know that Michelle was blind. It had just never 40 in conversation. When I
told my daughter that Michelle was blind, she was 41 for a moment. At first, she didn’t believe me.
“But Mommy, Michelle knew exactly what was in my picture!” ayla 42 . And I knew my child was 43
because Michelle had listened to ayla describe her art work. Michelle had also heard ayla’s 44 in her
work.
We were silent for a minute. Then ayla said slowly, “You know, Mommy, Michelle really did ‘see’ my
picture. She just used my 45 .”
Indeed, she uses a special type of “vision” that all mothers have.
26.A.senses B.means


B.found
B.except















C.methods
C.missed
C.since
C.she




D.ways
D.lost 27.A.enjoyed
28.A.instead
29.A.who
30.A.guests


D.but


D.which
D.friends
D.witnessed
B.it
B.family C.children
31.A.realized
32.A.sad
B.heard C.recognized
C.excited
C.a little










B.satisfied
B.not so

D.enjoyed
D.not at all
D.mix
D.played
D.encouragement
D.paper
33.A.especially
34.A.paint
35.A.stayed


B.draw




C.create
C.talked B.painted
36.A.excitement
37.A.attitude
38.A.touched
39.A.after
B.surprise C.delight
B.color C.picture


B.distinguished C.saw
C.until
D.understood
D.when
D.talked about
B.before
40.A.turned out
41.A.curious
42.A.cried
43.A.right


B.come up C.referred to
B.quiet



C.puzzled
C.complained
C.worried
C.pride





D.worried
D.informed
D.uncertain


D.description
D.pens
B.insisted
B.wrong
44.A.shortcomings B.difficulties
45.A.paper B.eyes C.hands
第三部分: 阅读理解(共25小题。每小题2分,满分50分)
阅读下列短文,从每题所给的 A、B.、C、D 四个选项中,选出最佳答案,并在答题卡上将该项涂黑。
A
A sense of humor is just one of the many things shared by Alfred and Anthony Melillo, 64-year-old
twin brothers from East Haven who made history in February 2002. On Christmas Eve, 1992, Anthony had


a heart transplant from a 21-year-old donor. Two days before Valentine’s Day in 2002, Alfred received a
19-year-old heart, marking the first time on record that twin adults each received heart transplants.
“I’m 15 minutes older than him, but now I’m younger because of my heart and I’m not going to
respect him,” Alfred said with a big smile, pointing to his brother while talking to a roomful of reporters,
who laughed frequently at their jokes.
While the twins knew that genetics(基因) might have played a role in their condition, they recognized
that their eating habits might have also contributed to their heart problems. “We’d put half a pound of
butter on a steak. I overdid it on all the food that tasted good, so I guess I deserved what I got for not
dieting properly.”
The discussion moved to Anthony’s recovery. In the five years since his heart transplant, he had been
on an exercise program where he regularly rode a bicycle for five miles, swam each day, and walked a
couple of miles. He was still on medication, but not nearly as much as Alfred, who was just in the early
stage of his recovery.
“Right now I feel pretty young and I’m doing very well,” Anthony said. “I feel like a new person.”
Alfred said his goal, of course, was to feel even better than his brother. But, he added, “I love my brother
very much. We’re very close and I’m sure we’ll do just fine.”
46. What did Alfred and Anthony have in common?
A. Exercise programs.
C. A sense of humor.


B. Education background.
D. Love for bicycling.
47. Alfred said he was younger than Anthony because _______.
A. he had a more successful heart transplant
B. he recovered faster from the transplant
C. he was born 15 minutes earlier than Anthony
D. his new heart was younger than Anthony’s
48. This article is mainly about _______.
A. the danger of heart transplanting
B. becoming young by getting a new heart
C. the effect of genetics on the heart
D. the twin brothers who received heart transplants
49. What did Alfred and Anthony think caused their heart problems?
A. Exercise. B. Diet. C. Laziness. D. Medicines.
50. What does the underlined part in Paragraph 3 refer to?
A. His heart problem. B. His new heart.


C. The attention he received. D. The food that tasted good.
B
Once I invited a group of friends round to my house, telling them that I was going to record their
speech. I said I was interested in their regional accents, and that it would take only a few minutes. Thus, on
one evening, three people turned up at my house and were shown into my front room. When they saw the
room they were a bit alarmed, for it was laid out as a studio. In front of each easy chair there was a
microphone at head height, with wires leading to a tape-recorder in the middle of the floor. I explained that
all I wanted was for them to count from one to twenty. Then we could relax and have a drink.
I turned on the tape-recorder and each in turn seriously counted from one to twenty in their best accent.
When it was over, I turned the tape-recorder off and brought round the drinks, and for the rest of the
evening there was general cheerful conversation—interrupted only by the fact that I had to take a
telephone call in another room, which unfortunately lasted some time.
Or at least that was how it would appear. For, of course, the microphones were not connected to the
tape-recorder in the middle of the room at all but to another one, which was turning happily away in the
kitchen. The participants, having seen the visible tape-recorder turned off, paid no more attention to the
microphones which stayed in front of their chairs, only a few inches from their mouths, thus giving
excellent sound quality. And my lengthy absence meant that I was able to obtain as natural a piece of
conversation as it would be possible to find.
I should add, perhaps, that I did tell my friends what had happened to them, after the event was over,
and gave them the choice of destroying the tape. None of them wanted to—though for some years
afterwards it always seemed to be my round when it came to the buying of drinks. Linguistic research can
be a very expensive business.
51. The writer asked his friends to count from one to twenty because _______.
A. he wanted to record the numbers for his research
B. he wanted to find out whether the tape recorder was working
C. he wanted to make his friends relax before real recording started
D. he wanted his friends to think that was all he wanted to record
52. Which of the following words can best describe the recording which the linguist managed to make?
A. controlled B. prepared C. natural D. artificial
53. The writer went into another room to ________.
A. get a natural recording of his friends’ conversation
B. stay away from too much drinking with his friends
C. bring a telephone into the front room
D. answer a long distance phone call
54. The writer turned off the tape-recorder because _______.
A. he had to answer a phone call
B. he wanted his friends to enjoy some drinks
C. he thought the tape-recorder might bother his friends
D. he wanted to make his friends believe he had finished the recording
55. The writer sounds _______ in telling the story.
A. serious B. humorous
C
C. honest D. excited


Even before my father left us, my mother had to go back to work to support our family. Once I came
out of the kitchen, complaining, “Mom, I can’t peel potatoes. I have only one hand.”
Mom never looked up from sewing. “You get yourself into that kitchen and peel those potatoes,” she
told me. “And don’t ever use that as an excuse for anything again!”
In the second grade, our teacher lined up my class on the playground and had each of us race across
the monkey bars, swinging from one high steel rod to the next. When it was my turn, I shook my head.
Some kids behind me laughed, and I went home crying.
That night I told Mom about it. She hugged me, and I saw her “we’ll see about that” look. The next
afternoon, she took me back to school. At the deserted playground, Mom looked carefully at the bars.
“Now, pull up with your right arm,” she advised. She stood by as I struggled to lift myself with my
right hand until I could hold the bar with my other elbow(肘). Day after day we practiced, and she praised
me for every rung(横档)I reached. I’ll never forget the next time, crossing the rungs; I looked down at the
kids who were standing with their mouths open.
One night, after a dance at my new junior high, I lay in bed sobbing. I could hear Mom come into my
room. “Mom,” I said, weeping, “none of the boys would dance with me.”
For a long time, I didn’t hear anything. Then she said, “Oh, honey, someday you’ll be beating those
boys off with a bat.” Her voice was faint. I peeked out from my covers to see tears running down her cheeks.
Then I knew how much she suffered on my behalf. She had never let me see her tears.
56. Which can be used to describe Mom’s attitude when she made the child peel potatoes?
A. Cruel. B. Favorable. C. Strict. D. Sympathetic.
57. When the author looked down at the kids, they were standing with their mouths open because_______.
A. they felt sorry for what they had done before
B. they were afraid the author might fall off and get hurt
C. they wanted to see what the author would do on the bars
D. they were astonished to find the author’s progress
58. What does the sentence “I saw her ‘we’ll see about that’ look” imply?
A. Mom believed every aim could be achieved if you stuck to it.
B. The race across monkey bars was not difficult enough for a child to give up.
C. Mom was determined to prove she herself was better than the teacher.
D. What the child had said brought Mom great attraction and curiosity.
59. From the passage, we know monkey bars can help a child train ______.
A. the strength and skill to hang and sway
B. the speed of one’s hand movement


C. the skill to throw and catch things
D. the bodily skill to circle round a bar

60. The most probable conclusion we can draw after reading the passage is ______.
A. the last incident was sad enough to make Mom weep
B. the child’s experience reminded Mom of that of her own
C. Mom could solve any problem except the one in the last paragraph
D. in fact Mom suffered more in the process of the child’s growth
D
Long bus rides are like television shows. They have a beginning, a middle, and an end with
commercials thrown in every three or four minutes. The commercials are unavoidable. They happen
whether you want them or not. Every couple of minutes a billboard (广告牌) goes by outside the bus
window. “Buy Super Clean Toothpaste.” “Drink Good Wet Root Beer.” “Fill up with Pacific Gas”. Only if
you sleep, which is equal to turning the television set off, are you free from the unending cry of “You Need
It! Buy It Now!”
The beginning of the ride is comfortable and somewhat exciting, even if you’ve traveled that way
before. Usually some things have changed new houses, new buildings, sometimes even a new road. The bus
driver has a style of driving and it’s fun to try to figure it out the first hour or so. If the driver is
particularly reckless (鲁莽的) or daring, the ride can be as thrilling as a suspense (悬疑) story. Will the
driver pass the truck in time? Will the driver move into the right or the left hand lane? After a while, of
course, the excitement dies down. Sleeping for a while helps pass the middle hours of the ride. Food always
makes bus rides more interesting. But you’ve got to be careful of what kind of food you eat. Too much salty
food can make you very thirsty between stops.
The end of the ride is somewhat like the beginning. You know it will soon be over and there’s a kind of
expectation and excitement in that. The seat of course, has become harder as the hours have passed. By now
you’ve sat with your legs crossed, with your hands in your lap, with your hands on the arm rest even with
your hands crossed behind your head. The end comes just at no more ways to sit.
61. According to the passage, what do the passengers usually see when they are on a long bus trip?
A. Buses on the road. B. Films on television.
D. Gas stations. C. Advertisements on the billboards.
62. What is the purpose of this passage?
A. To give the writer’s opinion about long bus trips.
B. To persuade you to take a long bus trip.
C. To explain how bus trips and television shows differ.


D. To describe the billboards along the road.
63. The writer feels long bus rides are like TV shows because ____________.
A. the commercials both on TV shows and on billboards along the road are fun
B. they both have a beginning, a middle, and an end, with commercials in between
C. the drivers are always reckless on TV shows just as they are on buses
D. both traveling and watching TV are not exciting
64. The writer of this passage would probably prefer ______________.
A. bus drivers who are reckless
C. a television set on the bus
B. driving alone
D. no billboards along the road
65. The writer thinks that the end of the ride is somewhat like the beginning because both are _________.
A. exciting B. comfortable
E
A sunflower is a sunflower. A mobile phone is a mobile phone. But can you combine the two to do
something for your local environment? As early as next year it may well be possible. When you have
finished with your mobile phone you will be able to bury it in a garden or a plant pot and wait for it to
flower.
A biodegradable (生物所能分解的) mobile phone was, this month, introduced by scientists. It is hoped
that the new type of phone will encourage consumers to recycle.
Scientists have come up with a new material over the last five years. It looks like any other plastic and
can be hard or soft, and is able to change shape. Over time it can also break down into the soil without
giving out any poisonous chemicals. British researchers used the new material to develop a phone cover
that contains a sunflower seed. When this new type of cover turns into waste, it forms fertilizers. These feed
the seed and help the flower grow.
Engineers have designed a small transparent (透明的) window to hold the seed. They have made sure it
only grows when the phone is thrown away. “We’ve only put sunflower seeds into the covers so far. But we
are working with plant experts to find out which flowers would perform best. Maybe we could put roses in
next time,” said one scientist.
As phone technology is developing so quickly, people are constantly throwing their mobiles away. This
means manufacturers are under pressure to find ways of recycling them. Some 650 million mobile phones
have been sold this year. Most of them will be thrown away within two years, adding plastic, heavy metal
and chemical waste to the environment. A biodegradable cover can offer some relief for nature, according
to the scientists.
“The seed comes out and the flower grows in the pot so you don’t have to concern yourself with the
C. tiring D. boring


phone when you have finished using it,” said erry ieran. She leads the research team, which is based at the
University of Warwick in Britain.
66. What is the purpose of this passage?
A. To tell the popularity of biodegradable cell phones.
B. To persuade the reader to buy the biodegradable cell phone.
C. To discuss the development of phone technology.
D. To introduce an environmentally-friendly cell phone to readers.
67. It could be learned from the passage that _________.
A. developing the new type of phone is mainly to protect the environment
B. phone-makers will benefit much more from the new type of phone
C. the new type of phone will certainly be popular with users all over the world
D. the phones that can be recycled are available only in Britain now
68. People throw away their cell phones most probably because _______.
A. there is something wrong with them
B. no sunflowers can grow out of them
C. they are out of fashion
D. they are becoming cheaper and cheaper
69. What might be the best title for this passage?
A. Sunflower and Phone

B. Plant Your Phone
D. No Worry about Phone C. Protect Our Environment
70. Where can we probably find the passage above?
A. In a novel B. In a magazine C. In a textbook
第Ⅱ卷(共55分)
第四部分 书面表达(共三节,满分55分)
D. In a diary


第一节: 根据汉意 补全句子,答案写在答题纸上(共10小题,每小题1分,满分10分)

71. 我们应该充分利用我们的时间来提高成绩。
72. 周苏红在中国女排中起着重要作用。
73. 这个新政策是否能够实施有待观察。
74. 站在高山上,呼吸会非常困难。
75. 因为糟糕的天气,比赛不得不推迟。
76. 当处在困境时你容易灰心吗?
77. 只有在他为自己的无礼道歉时,我才会再给他说话。
78. 在我看来:毫无疑问,北京奥运会是历史上最好的一次。
79. 联合国高度赞扬中国在维护世界和平所起的作用。
80. 从他所说的来判断,他没有把各种因素都考虑进去。
第二节 阅读表达(共5小题;每小题3分,满分15分)
阅读下面短文并回答问题, 然后将答案写到答题卡相应的位置上(请注意问题后的词数要求)。
[1]One person can make a difference. Just ask Ryan Hreljac, who is 15 and lives in Canada. Without
his help, hundreds of wells that now provide fresh water for people in Africa, Central America, and India
might never have been built. He travels the world to tell people how they can help solve a big problem
_______ in many developing countries.
[2] In 1998, when he was 6 years old, Ryan learned that children in Africa often must walk miles each
day to find water. Some even die from drinking bad water in some developing countries, his teacher says. So
Ryan decided to act. He spoke to schools, churches, and clubs about his goal. The word spread, and
donations began coming in. After several months of hard work, Ryan had raised $$2,000, enough to dig one
well. An organization called Canadian Physicians for Aid and Relief chose a location for the well in the
village of Agweo in Africa.
[3] After the first well, “the ripple effect took over,” Ryan says, “and one goal led to another.” Its


website (ht help one more person.”
[4] Asked if he ever feels discouraged that many people still lack good water, Ryan says, “It’s important
to be an optimist. I had a small dream, and I stayed with it. Everybody can do something.”
81. What’s the main idea of Paragraph 2? (no more 10 words)
82. Explain the underlined sentence in Paragraph3.
83. For what purpose does Ryan accept awards? (no more 15 words)
84. According to what Ryan said in Paragraph4, what is Ryan’s attitude toward his work? (no more 6
words)
85. Fill in the blank in Paragraph1 with proper words. (no more 8 words)
第三节 写作(满分30分)
Dear Peter,
I am a senior high school student in the second grade. I am extremely busy with my school these days
and yet things don’t go well. I know it is very important to study because the college entrance examination
is coming next year. How can I deal with the boring and stressful life? Hope to get help from you.
Yours,
Sonia
Hi Sonia,
The first thing I want to tell you is that you are not alone. Millions of other students have felt this way
and many thousands feel this way right now.
……………..
Good luck and please let me know what happens.
Yours,
Peter











单项填空 DBCCA CBBBD AACAB ABAAC BCADA
完型填空ACBAA ACADB BCDCB BBACB
阅读理解CDDBA DCADB CDAAD CABDA DACBB
翻译句子 full use ofmake the most ofmake the best of improve
72. playing an important partrole
73. to be seen whether
74. on top hard to breathe
75. put off because ofdue to owing to
76. lose heart in trouble77. when he apologizes 78. There is no doubt that
79. thinksthought highlymuchof
80. Judging fromby, take …into considerationaccount
阅读表达:81. How Ryan raised money to build the first well. How Ryan collected money to build the first
well. How Ryan achieved his first goal.
82.. After the first well, many wells were built. One goal led to another.
83.. What he says when he gets awards might help more people. Each word he says when he gets one award
might help one more person.
84. He is optimistic about it.
85. the lack of safe drinking water many people still lack good water many people can’t get fresh water
写作: One possible version
Hi Sonia,
The first thing I want to tell you is that you are not alone. Millions of other students have felt this way
and many thousands feel this way right now.
Since you already know your grades are important, you should understand what you are doing right
now will have a large influence on how well you will do next year. So think about what you want to do with
your future and try to make your dream come true. That’s the long-term reward which may help to give
you the push to study hard now.
That doesn’t mean, however, that you can’t have some fun now. You still have your weekends and
sometime after school. Then the answer is to plan your days. Make a timetable for each day that shows you
what time you must use for school and what time you can have for relaxation. Things like playing outdoor
games, watching videos or chatting with your friends can surely make you feel refreshed.


The school year won’t last forever, my friend, and you will be out there in the sunshine before you
know it.
Good luck and please let me know what happens.
Yours,
Peter


高考模拟英语试卷
考试时间:120分钟 试卷满分:150分
第Ⅰ卷(选择题 共100分)
第一部分 听力(共两节,满分30分)
第一节(共5小题,每小题1.5分,满分7.5分)
听下面5段对话。每段对话后有一个小 题,从题中所给的A、B、C三个选项中选出最佳选项,并标
在试卷的相应位置。听完每段对话后,你都 有10秒钟的时间来回答有关小题和阅读下一小题,每段对话
仅读一遍。
例:How much is the shirt?
A. £19.15.
答案是 B。
1. What does the man prefer to do on Sundays?
A. Go shopping. B. Go swimming. C. Do some reading.
B. £9.15. C. £9.18.
2. Where does the conversation probably take place?
A. In a hotel. B. In a hospital. C. In a restaurant.
3. When will Professor Davidson talk with the woman?
A. After his class. B. The next day. C. Before office hours.
4. Where will the man sit in the restaurant?
A. Near the door. B. In the corner. C. Near the window.
5. Why will the woman go to the park today?
A. To play basketball. B. To play volleyball. C. To walk with her friends.
第二节(共15小题,每小题1.5分,满分22.5分)
听下面5段对话或独白。每段对话 或独白后有几个小题,从题中所给的A、B、C三个选项中选出最
佳选项,并标在试卷的相应位置。听每 段对话或独白前,你将有时间阅读各个小题,每小题5秒钟;听完
后,各小题将给出5秒钟的作答时间。 每段对话或独白读两遍。
听第6段材料,回答第6-7题。
6. Why is Harry unwilling to join the woman?
A. He wants to watch TV. B. He has a pain in his knee. C. He is too lazy.
7. What will the woman probably do next?
A. Stay at home. B. Play with Helen. C. Take Harry to hospital.
听第7段材料,回答第8-9题。
8. Where did the man possibly lose his bag?


A. On the flight. B. At the airport. C. In the hotel.
9. What color is the basketball toy?
A. Black. B. Brown. C. Blue.
听第8段材料,回答第10-12题。
10. How long has the man been in the hospital?
A. For 2 years. B. For 6 years. C. For 7 years.
11. What will the man serve as in Nigeria?
A. An organizer. B. A nurse. C. A doctor.
12. What can we learn from the conversation?
A. The man got his new job this morning.
B. The woman is against the man's decision.
C. The man wants to marry Rosie.
听第9段材料,回答第13-16题。
13. What would Joe probably do during the Thanksgiving holiday?
A. Go to a play. B. Visit ingston. C. See a car show.


14. What is Ariel going to do in Toronto?
A. Attend a party. B. Meet her aunt. C. Stay at home.
15. Why is Ariel in a hurry to leave?
A. To call up Betty. B. To pick up Daniel. C. To buy some DVDs.
16. What might be the relationship between the speakers?
A. Classmates. B. Fellow workers. C. Guide and tourist.
听第10段材料,回答第17-20题。
17. What must you do for the English course?
A. Take a test. B. Finish the previous level.
C. Study a specific culture.
18. Why is history open at all levels?
A. Students have different levels.
B. Different teachers teach courses.
C. Cultures and time periods vary.
19. What is included in the optional courses?
A. History. B. Science. C. English.


20. What will the speaker do for the students?
A. Help them arrange their courses.


B. Help them sign up for a course.
C. Help them find room in a course.

第二部分 阅读理解(共两节,满分40分)
第一节(共15小题;每小题2分,满分30分)
阅读下列短文,从每题所给的四个选项(A、B、C和D)中选出最佳选项,并在答题卡上将该项涂
黑。
A
Qibao Ancient Village
Qibao, located in the center of Minhang District, Shanghai, is a village with a history of one thousand
years. In the village, wine and tea are served on old- fashioned square tables together with long benches,
long-mouth copper pots and flat-end chopstick used. The most famous snacks in old Shanghai are square
pastry, rice wine and steamed salted pork in wine.
Telephone 021-25
Entrance Ticket 45 yuan per all-in-one ticket (preferable price of 30 yuan is available now), covering
almost all tourist attractions inside the village.
Jinshan Village of Farmer Painting
Villagers skillfully make good use of folk arts such as printing and dyeing, embroidery (刺绣), wood
carving. They take the various folk customs and the busy scenes of labor of villagers in the lower
Changjiang valley as the theme of paintings and create farmer paintings in a simple style.
Telephone 021-55
Entrance Ticket 30 yuanperson
Merry Countryside Tour in honghua Village
The village provides tourists with accommodations, tours, chess, cards, fitness and entertainment
through renting out separate farmhouse and sells tourist products and agricultural by-products related to
the merry countryside tour.
Telephone 021-33
Entrance Ticket Free
Pudong Lingkong Agric Gardening
It is one of Shanghai countryside tour scenic spots, which features art of teapots, crop plantation and
export. The Geological Science Popularization Hall stores up tens of thousands of rare stones collected all
over the world.
Telephone 021-57


Entrance Ticket 50 yuanperson
21. Which tour is offering a discount on the entrance ticket now?
A. Qibao Ancient Village.
B. Jinshan Village of Farmer Painting.
C. Merry Countryside Tour in honghua Village.
D. Pudong Lingkong Agric Gardening.
22. Pudong Lingkong Agric Gardening may especially attract people who are interested in______.
A. traditional snacks B. folk arts C. farming D. rare stones

23. What is the theme shared by the four tours?
A. Countryside life.







B. Agricultural achievements.
D. Traditional customs. C. City developments.

B

What is the most important day of your life? For many people the answer is your Wedding Day—the
day when you marry another person and promise to live together as husband and wife for the rest of your
lives.
Wedding celebrations differ from country to country—in China the bride wears red while in India the
wedding continues for three days. However, in Britain the bride wears white and wedding usually only lasts
for one day.
In the U it is possible to get married in a religious or civil ceremony. A religious ceremony takes place
in a church. A civil ceremony, on the other hand, can be held in an office, in a hotel, or even on a boat.
On the day of the wedding the bride and groom are kept separate until the ceremony. It is said to be
bad luck if the groom sees the bride in her dress before the wedding. The bride arrives at the ceremony
accompanied by her father and bridesmaids. They officially give her away to her new husband. During the
ceremony the happy couple exchange vows and give each other wedding rings which they will wear forever.
At the end of the ceremony they kiss.
Afterwards they go outside where friends and family throw rice or color papers over them and then
they go on to the reception where there is a lot of food and drink. After the meal the father of the bride and
the best man make speeches. Then the bride and groom take the first dance. It is a lot of fun. Finally, at the
end of the party the newly-weds leave to go on honeymoon, usually to a very romantic destination.
24. What function is the first two paragraphs?
A .To tell us that we have most important day in our life.


B. To tell us Wedding day is the day when you marry another person and promise to live together.


C. To tell us that people hold wedding.
D. To attract reader's attention to the topic weddings in Britain.
25. What does the underlined word ‘newly-weds' (Para 5) mean?
A. The bridegroom.

B. The bride.
D. The best man.

C. The newly-married couple.
26. What can we infer from the passage?
A. People all over the world pay a great deal attention to Wedding Day.
B. In Britain the bride wears white.
C. Wedding celebrations differ from country to country.
D. In India the wedding can continue for three days.
27. Which of the following statements is NOT true?
A. People in Britain usually get married in a religious or civil ceremony.
B. On the wedding day, the bridegroom can't see the bride in a dress.
C. The bride's father will give her away to the bridegroom.
D. The bride and bridegroom will take the first dance.

C

Humans have launched themselves into the outer space. They've landed on the moon. They've built
habitable space stations that orbit the Earth. The next giant leap for mankind is to reach another planet–
specifically, Mars.
The problem is that it's no easy task. The planet is 586 times further away from the Earth than the
moon, and it'll take around 180 to 220 days to reach Mars, depending on where each planet is in its orbit.
Such long periods in space have suggested many potential health problems, including hormonal changes,
skin conditions, and muscle and bone deterioration (损耗).
Here's where some furry friends come in. A wide range of animals have been in space, from fruit flies
and spiders to cats, and dogs. Such experiments began as far back as the late 1940s in first tests to see if
living things could withstand the extreme g-force (重力) of a rocket launch.
Mice continue to play a very important part in space experiments, mainly because the animals make
excellent test subjects. They're small, which makes them inexpensive and easy to care for. In addition, their
size and short life span make it possible to do the equivalent of several human years of tests in a much
shorter time. Finally, because mice are mammals, they share many common characteristics with humans in


terms of genetics, biology and behavior.
Astromice have hit the headlines recently, as a team of scientists led by Betty Nusgens, professor of
biology at the University of Liege in Belgium, found that the mice suffered a 15 percent thinning of their
skin after 91 days aboard the International Space Station (ISS).
This experiment was part of a wider NASA mission (任务) called the Mice Drawer System (MDS). The
Italian Space Agency developed the facility, which allows six mice to be housed, monitored, and
automatically fed and watered aboard the ISS, among which three survived during the mission.
The mice have participated in 20 separate experiments, to study such effects as osteoporosis (骨质疏松
症), anemia (贫血) and heart health.
Results for the 20 experiments are coming in gradually. But it's clear that mice continue to play an
important role in the ongoing quest to conquer the final frontier.
can infer from Paragraphs 2-3 that ______.
A. Mars is the farthest planet away from Earth discovered so far
B. animals that have been sent into space have mostly survived
C. it was in the late 1940s that animals were first sent to the ISS
D. he journey to Mars could put humans' health at risk
underlined word
A. set up
C. work out
B. hold up
D. come across
ing to Betty Nusgens and her team, the mice aboard the ISS ______.
A. suffered the loss of part of their skin
B. all survived for the duration of the mission
C. were fed and watered by the astronauts
D. participated in 20 experiments that made great breakthroughs
main purpose of the article is to _____.
A. describe the role mice play in scientific research
B. report on the results of the Mice Drawer System
C. analyze how mice could pave the way to Mars
D. change people's traditional attitudes toward mice

D

WELLINGTON, New ealand (AP) ---- About 140 pilot whales that stranded(使...搁浅) themselves on a


remote stretch of New ealand beach have died, but conservation workers and volunteers are hoping the
remaining 60 or so will survive after they managed to get them refloated, an official said Saturday.
The geography of Farewell Spit on the South Island seems to work against whales, which regularly
become stranded there. The stranding of 198 whales on Friday was one of the largest in recent years and
inspired 80 workers and volunteers to help out.
By late Saturday, 140 of the whales had died, said Department of Conservation area manager Andrew
Lamason.
He said the workers and volunteers worked throughout the day to keep the surviving whales watered
and covered before refloating them during Saturday night's high tide.

country,
He said the surviving whales had moved to deeper water. But he cautioned that although hopes for
their survival were high, whales had been refloated in the past only to return and get stranded again.
Lamason said the scale(规模) of the stranding has been tough on the helpers, both physically and
mentally.

The focus will now turn to the dead whales, and the enormous task of dealing with the remains,
Lamason said, adding that in the past helpers have typically buried them in the sand, but that he's not sure
what the approach will be this time.
He said the department had been experimenting with moving the the dead whales into the water, which
has the advantage of providing food for other sea creatures.
did the volunteers have to wait until Saturday before they could refloat the whales?
A. Because they were very busy on Friday.
B. Because they had to wait for the sea water to rise.
C. Because the whales were too intelligent.
D. Because the whales were buried deep in the sand.
of the following words can be used to describe Lamason's attitude towards the result of refloating
the whales?
A. Doubtful.
C. Positive.


B. Optimistic.
D. Careless.
did Lamason say about the feelings of the workers and volunteers?
A. They were very happy to see so many whales.
B. They thought refloating the whales was not a hard job.


C. They were physically tired and emotionally hurt.
D. They believed that this is very natural.
will the volunteers deal with the dead whales this time according to Lamason?
A. They will bury them in the sand.
B. They will throw them into the ocean.
C. They will feed them to other sea creatures.
do not have any exact idea yet.

第二节(共5小题;每小题2分,满分10分)
根据短文内容,从短文后的选项中选出能填入空白处的最佳选项,选项中有两项为多余选项。
Nowadays more and more children chat daily either online or through their mobile phones. 36
Some are shy
people of their own age on relationship issues, or problems at home.
Sometimes the online world, just like the real world, can introduce problems, such as bullying or
arguments. 37 But there are also a few people who use the Internet for offensive (冒犯性的) or
illegal aims. Children must be made aware of both the good things and the dangers.
38 Just as you decide which TV programs are suitable, you need to do the same for the websites
and chat rooms your children visit. Remind your children that online friends are still strangers. Reminding
them of the risks will keep them alert (警惕).
39 So it's likely that your children may know more than you do. We get left behind when it
comes to the latest gadgets and the interactive areas of websites, like chat rooms and message boards, which
are especially strange. 40 Chatters love to use abbreviations (缩写) such as atb—all the best, bbfn
—bye bye for now, gr8—great, Idk—I don't know, kit—keep in touch, paw—parents are watching, lol—
laugh out loud. It seems like another language, and it is!








第一节 完形填空(共20小题;每小题1.5分,满分30分)
阅读下面短文,从短文后各题所给的四个选项(A、B、C、D)中,选出可以填入空白处的最佳选项,
并在答题卡上将该项涂黑。
A Toronto man is offering a free round-the-world air to the right woman. But __41___ apply. You must
be named Elizabeth Gallahgher and have a Canadian ___42___ .
Jordan Axani, 28, said he and his then girlfriend, Elizabeth Gallagher, booked heavily discounted
A. Going online is great fun.
B. Computer studies are part of schoolwork now.
C. The language of chat is strange to many parents, too.
D. There are some websites that are not suitable for children.
E. To keep children safe, your management must cover the family computer.
F. They are connecting to a huge number of other children all over the world.
G. Surfing the Internet takes too much of the time that should be spent on lessons.


round-the-world air tickets in May, but their ___43___ ended and he did not want her ticket to ___44___.
The ticket had a strict no-transfer(不可转让)___45___, but since passport information was not required
when ___46___ , any Canadian Elizabeth Gallagher can___47___ it.
“I just want to see the ticket go to good use and for someone to ___48___ a lot of joy,” said Axani. He
posted his ___49___ on a social networking website, and received thousands of e-mails, including thirty
from actual Elizabeth Gallagbers with the ___50___ passports, “More ___51___ , there are hundreds of
Canadians who are interested in ___52___ their name to Elizabeth Gallagher,” Axani said. “It was
absolutely out of ___53___ , thousands of e-mails, people around the world __54___ their stories of travel.”
Axani wrote in his post that he is not ___55___ anything in return and that the woman who uses the
___56___ ticket can choose to either travel with him or __57___ the ticket and travel on her own.
The ___58___ is scheduled to start on December 21 in New York City and continue on to Milan,
Prague, Paris, Bangkok and New Delhi before ___59___ in Toronto on January 8. He said the ___60___
woman will be announced on the website and the trip will be shared online.
41.A. benefits B. deposits C. restrictions D. examinations
42.A. origin B. passport C. accent
C. dream
C. go no sale

D. Friend
D. relationship
D. come into effect
43.A. holiday B. marriage
44.A. go to waste B. come to mind
45.A. policy B. order C. payment D. schedule
46.A. applying B. checking C. booking D. bargaining
D. buy 47.A. choose B. borrow C. use
48.A. sacrifice B. experience
49.A. answer B. offer
50.A. right
C. express D. provide
D. comment
D. real
C. advice
B. same C. now
51.A. convincing B. annoying C. satisfying
52.A. changing
53.A. touch
B. giving
D. interesting
C. lending D. writing
D. date B. question C. control
54.A. admiring B. sharing C. advertising D. doubting
D. looking for
D. extra
55.A. leaving B. dealing with C. losing
56.A. single B. strange C. regular
B. take 57.A. return
58.A. interview
C. reserve D. hide
D. meeting
D. staying
D. lucky
B. program C. trip
59.A. ending B. calling
60.A. honored B. lovely
C. repeating
C. intelligent



第Ⅱ卷(非选择题 共50分)
第二节 语法填空(共10小题;每小题1.5分,满分15分)
阅读下面材料,在空白处填入适当的内容(不多于3个单词)或括号内单词的正确形式。
I was traveling in a small place in New York. That place had no taxi for a short ___61___(distant).
One had to take a rickshaw(人力车).
I called out for an old man in the waiting line. Before I got on the rickshaw, I ___62___(automatic)
asked how much he would charge to take me across to the Mall. I ___63___(tell) dollarsThen I
started bargaining. He just gave a hard smile and said,
necessary 64 (bargain) on this small amount.
I was a bit ashamed at myself and kept quiet while he continued, 65 (like) those taxis, my
vehicle does not run on petrol or diesel(柴油) or even gas; 66 runs on my sweat. So please give it to me
67 a smile, for the money has to feed so many ___68 (hunger) mouths at home.
When we reached the Mall, I gave him the money and the smile 69 requested and added a gentle
pat on his back. He smiled back as if to give me a receipt and I would preserve his smile for 70 long
time.


第四部分:写作(共两节 满分35分)
第一节 : 短文改错(共10小题;每小题1分,满分10分)
增加: 在缺词处加一个漏字符号( ∧ ),并在其下面写出该加的词。
删除: 把多余的词用斜线()划掉。
修改: 在错的词下划一横线,并在该词下面写出修改后的词。
注意: 1. 每处错误及其修改均仅限一词;
2. 只允许修改10处,多者(从第11处起)不记分。

Women are playing a much important part in society than before. Now women are working as teachers,
scientists but even leaders. Almost all jobs that used to be done by men is done perfectly well by women.
Women are no longer looked down in society. As the changes in their social role, women's position in the
family has been improved as well. That is hard to find the wife is busy while the husband is sitting in an
armchair, watches TV. Though there are a lot of changes, the great number of men still guard their rights.
Sometimes few women are allowing to attend important meetings. This is the problem we should try to
solve it now.



第二节: 书面表达(满分25分)

2.结合实际从以下三方面提出具体建议:
a.膳食; b.体育锻炼; c.人生态度。
注意:
1.词数100左右;
2.可以适当增加细节,以使行文连贯;




________________________________________________ ___________________________________________
___ __________________________________________________ __
___________________________________________ ______________________________
_______________ __________________________________________________ ________
_____________________________________ ____________________________________
_________ __________________________________________________ ________________________________
______________ _________________________________________
____ __________________________________________________ ___________________
__________________________ _______________________________________________ ________________________________________________ _________________________
____________________ __________________________________________________ ___
__________________________________________ _______________________________


听力:
1-5 CABCB 6-10 ABBBB 11-15 CACAB 16-20 BACBA

阅读:
(A ) ADA (B) DCAB ( C )DBAC ( D )BACD


七选五
FAEBC

完形:
41—45 CBDAA 46—50 CCBBA 51—55 DACBD 56—60 DBCAD

语法填空:
61.distance
62.automatically
63.was told
64.to bargain
65.Unlike
66.it
67.with
68.hungry
69.as
70.a

改错
【小题1】much 改为more
【小题2】but改为 and
【小题3】 is 改为 are
【小题4】down后加onupon
【小题5】 As 改为 With
【小题6】That与 It
【小题7】watch 改为 watching
【小题8】 the number of是“…的数量”, a number of很多
【小题9】allowing 改为 allowed
【小题10】it删除

书面表达
Recently I find that some classmates lead an unhealthy life, such as staying up late and
not eating breakfast. Although we are under great pressure, it is necessary for us to live a


healthy life. The following are some useful tips.
First, it’s important that we have a balanced diet and enjoy breakfast every day, because
we consume much energy while studying. Second, taking exercise regularly is of equal importance,
which can help us to keep fit and release stress. More importantly, we should develop a positive
attitude, which allows us to look at things positively no matter what happens.
In a word, proper food, regular exercise and a positive attitude can be beneficial to us
during this bitter but sweet year.


高考模拟英语试卷
第I卷
第二部分 阅读理解(共两节,满分40分)
第一节 (共15题;每题2分,满分30分)
阅读下列短文,从每题所给的四个选项(A、B、C和D)中,选出最佳选项。
A
NOTTINGHAM FORUM(论坛)
GEOFF HOON
“__________(24)”
Geoff Hoon is one of Britain’s most experienced politicians.
Mr Hoon joined the Foreign and Commonwealth office in May 1999 as Minister of State with
responsibility for Asia, the Pacific, the Middle East and North Africa; he was then made Minister
for Europe. Later, in 1999, Mr Hoon joined Tony Blair’s Cabinet(内阁)as Secretary of State
for Defence, where he oversaw four majordeployments(部署)of British troops, including to Iraq.
After the 2005 general election he became leader of the House of Commons and earlier this year
(2006) he was appointed as Minister for Europe again.
On Friday 24 November Mr Horn is visiting Nottingham to address Forum. He will speak about
his experience in the public spotlight, and explain what life as a Cabinet Minister is really
like.
There will be an opportunity for questions after the talk, which is open to all students
and staff at NottinghamUniversity. Please arrive early as there are limited spaces and nobody
will be admitted once the event had begun.
FRIDAY 24 NOVEMBER
4 O’LOC
A48 SIR CLIVE GRANGER (GEOGRAPHY & ECONOMICS)
FREE ENTRY

21. At the time of the writing, Geoff Hoon was a ______.
A. retired politician B. university scholar C. state
TH
th
leader D. military consultant
22. Whoever wanted to attend were advised to “arrive early” in order ______.
A. to be allowed to ask questions B. to be


admitted to the talk
C. not to interrupt the speaker D.
to find a place closer to the speaker
23. By “A48 SIR CLIVE GRANGER”, the writer gives information about ______.
A. who was organizing the event B. where the event was
to take place
C. how one could get admitted D. how old the
lecturer was
24. The topic of the talk missing in the 3 line (blank) should be ______.
A. Life in the
rd
Cabinet B. An Excellent
Lecturer
C. Top State Secrets to Be Let Out D. Analysis of World
Situation
B
Georgia Aquarium is open 365 days a year. Please note that hours of operation do vary.Last
ticket sold and entry is 30 minutes before closing time.
Regular Hours
Sunday—Friday:10 a.m.—5 P.m.
Saturday:9 a.m.—6 p.m.
Summer Hours:May 27-August 16
Monday-Saturday:9 a.m.—9 P.m.
Sunday:9 a.m.—6 p.m.
Full Price Total Ticket
Adult(ages 13-64):$$38.95
Child(ages 3-12):$$32.95
Senior(age 65+):$$34.95
Total Ticket Online Discount
Adult(ages 13-64):$$35.95
Child(ages 3-12):$$29.95
Senior(age 65+):$$31.95
Animal Encounters:$$59.95
Beluga&Friends Interactive Program:$$179.95


Journey with Gentle Giants Immersion Program:$$234.95
AT&T Dolphin Tales
Our new dolphin gallery, theater and show are now open.The gallery and viewing window are
open to general admission guests periodically throughout the day.Show times vary each day, so
check for times when you are purchasing general admission tickets.Don’t forget- AT&T Dolphin
Tales is included in Total Ticket general admission,so don’t miss this spectacular show !
4D Theater—Deepo’s Undersea 3D Wondershow Presented by AT&T
With multiple shows throughout the day,be sure to catch this unique experience at Georgia
Aquarium.Admission to the 4D Theater is included in Total Ticket general admission!
Journey with Gentle Giants Immersion Program
Swim or scuba dive in the Ocean Voyager exhibit,built by The Home Depot.Swims are daily
at 4:30 p.m.,and scuba dives are daily at 3 p.m.and Saturday and Sunday at ll a.m.Spots
are limited,so it’s best to book in advance.
Beluga&Friends Interactive Program
Interact with belugas and harbor seals in the Georgia-Pacific Cold Water Quest gallery.This
animal interaction occurs daily at 10 a.m.and 1:30 p.m.Spots are limited,so it’s best
to book in advance.
Animal Encounters
Interact with a penguin daily at l1:30 a.m.and l:45 p.m.You can also experience a special
encounter with a dolphin.Spots are limited,so it’s best to book in advance.
25.How much can they save if grandparents in their seventies with a 10-year-old boy visit
Georgia Aquarium with online tickets?
A.$$3. B.$$6. C.$$9. D.$$12
26.Which of the following about AT&T Dolphin Tales is TRUE?
A.It is open all day long to visitors.
B.It is really worth watching for visitors.
C.Visitors have to pay extra money to watch it.
D.visitors should book their tickets in advance.
27.With a Total Ticket,visitors are allowed to visit________.
A.Animal Encounters
B.Beluga&Friends Interactive Program
C.4D Theater-Deepo’s Undersea 3D Wondershow
D.Journey with Gentle Giants Immersion Program


C
How Do You See Diversity?
As a manager, Tiffany is responsible for interviewing applicants for some of the positions
with her company. During one interview, she noticed that the candidate never made direct eye
contact. She was puzzled and somewhat disappointed because she liked the individual otherwise.
He had a perfect résumé and gave good responses to her questions, but the fact that he never
looked her in the eye said “untrustworthy,” so she decided to offer the job to her second choice.
“It wasn’t until I attended a diversity workshop that I realized the person we passed
over was the perfect person,” Tiffany confesses. What she hadn’t known at the time of the
interview was that the candidate’s “different” behavior was simply a cultural
misunderstanding. He was an Asian-American raised in a household where respect for those in
authority was shown by averting (避开) your eyes.
“I was just thrown off by the lack of eye contact; not realizing it was cultural,” Tiffany
says. “I missed out, but will not miss that opportunity again.”
Many of us have had similar encounters with behaviors we perceive as different. As the world
becomes smaller and our workplaces more diverse, it is becoming essential to expand our
understanding of others and to reexamine some of our false assumptions.
Hire Advantage
At a time when hiring qualified people is becoming more difficult, employers who can eliminate
invalid biases (偏见) from the process have a distinct advantage. My company, Mindsets LLC, helps
organizations and individuals see their own blind spots. A real estate recruiter we worked with
illustrates the positive difference such training can make.
“During my Mindsets coaching session, I was taught how to recruit a diversified workforce.
I recruited people from different cultures and skill sets. The agents were able to utilize their
full potential and experiences to build up the company. When the real estate market began to
change, it was because we had a diverse agent pool that we were able to say in the real estate
market much longer than others in the same profession.”
Blinded by Gender
Dale is an account executive who attended one of my workshops on supervising a diverse
workforce. “Through one of the sessions, I discovered my personal bias,” he recalls. “I learned
I had not been looking at a person as a whole person, and being open to differences.” In his
case, the blindness was not about culture but rather gender.
“I had a management position open in my department; and the two finalists were a man and
a woman. Had I not attended this workshop, I would have automatically assumed the man was the
best candidate because the position required quite a bit of extensive travel. My reasoning would
have been that even though both candidates were great and could have been successful in the
position, I assumed the woman would have wanted to be home with her children and not travel.”


Dale’s assumptions are another example of the well-intentioned but incorrect thinking that
limits an organization’s ability to tap into the full potential of a diverse workforce.
“I learned from the class that instead of imposing my gender biases into the situation,
I needed to present the full range of duties, responsibilities and expectations to all candidates
and allow them to make an informed decision.” Dale credits the workshop, “because it helped
me make decisions based on fairness.”
Year of the now-It-All
Doug is another supervisor who attended one of my workshops. He recalls a major lesson learned
from his own employee.
“One of my most embarrassing moments was when I had a Chinese- American employee put in a
request to take time off to celebrate Chinese New Year. In my ignorance, I assumed he had his
dates wrong’, as the first of January had just passed. When I advised him of this, I gave him
a tong talking-to about turning in requests early with the proper dates.
“He patiently waited, then when I was done, he said he would like Chinese New Year off,
not the Western New Year. He explained politely that in his culture the new year did not begin
January first, and that Chinese New Year, which is tied to the lunar cycle, is one of the most
celebrated holidays on the Chinese calendar. Needless to say, I felt very embarrassed in assuming
he had his dates mixed up. But I learned a great deal about assumptions, and that the timing
of holidays varies considerably from culture to culture.
“Attending the diversity workshop helped me realize how much I could learn by simply asking
questions and creating dialogues with toy employees, rather than making assumptions and trying
to be a know-it-all,” Doug admits. “The biggest thing I took away from the workshop is learning
how to be more ‘inclusive’ to differences.”
A Better Bottom Line
An open mind about diversity not only improves organizations internally, it is profitable
as well. These comments from a customer service representative show how an inclusive attitude
can improve sales. “Most of my customers speak English as a second language. One of the best
things my company has done is to contract with a language service that offers translations over
the phone. It wasn’t until my boss received Mindsets’ training that she was able to understand
how important inclusiveness was to customer service. As a result, our customer base has
increased.”
Once we start to see people as individuals, and discard the stereotypes, we can move
positively toward inclusiveness for everyone. Diversity is about coming together and taking
advantage of our differences and similarities. It is about building better communities and
organizations that enhance us as individuals and reinforce our shared humanity.
When we begin to question our assumptions and challenge what we think we have learned from
our past, from the media, peers, family, friends, etc., we begin to realize that some of our
conclusions are flawed (有缺陷的) or contrary to our fundamental values. We need to train


ourselves to think differently, shift our mindsets and realize that diversity opens doors for
all of us, creating opportunities in organizations and communities that benefit everyone.
28. Tiffany’s misjudgment about the candidate stemmed from _______.
A. racial stereotypes
B. invalid personal bias
C. cultural ignorance
D. emphasis on physical appearance
29. What kind of organization is Mindsets LLC?
A. A real estate agency.
B. A personnel training company.
C. A cultural exchange organization.
D. A hi-tech company.
30. After one of the workshops, account executive Dale realized that ____.
A. he had hired the wrong person
B. he could have done more for his company
C. he had not managed his workforce well
D. he must get rid of his gender bias
31. What did Dale think of Mindsets LLC’s workshop?
A. It was well- intentioned but poorly conducted.
B. It tapped into the executives’ full potential.
C. It helped him make fair decisions.
D. It met participants’ diverse needs.
D
That’s enough, kids
It was a lovely day at the park and Stella Bianchi was enjoying the sunshine with her two
children when a young boy, aged about four, approached her two-year-old son and pushed him to
the ground.
“I’d watched him for a little while and my son was the fourth or fifth child he’d shoved,”
she says.” I went over to them, picked up my son, turned to the boy and said, firmly, ’No,
we don’t push,” What happened next was unexpected.
“The boy’s mother ran toward me from across the park,” Stella says,” I thought she was
coming over to apologize, but instead she started shouting at me for disciplining her child,
All I did was let him know his behavior was unacceptable. Was I supposed to sit back while her
kid did whatever he wanted, hurting other children in the process?”
Getting your own children to play nice is difficult enough. Dealing with other people’s
children has become a minefield.


In my house, jumping on the sofa is not allowed. In my sister’s house it’s encouraged.
For her, it’s about kids being kids”If you can’t do it at three, when can you do it?”
Each of these philosophies is valid and, it has to be said, my son loves visiting his aunt’s
house. But I find myself saying “no” a lot when her kids are over at mine. That’s O between
sisters but becomes dangerous territory when you’re talking to the children of friends or
acquaintances.
“ids aren’t all raised the same,” agrees Professor Naomi White of Monash University.”
But there is still an idea that they’re the property of the parent. We see our children as an
extension of ourselves, so if you’re saying that my child is behaving inappropriately, then
that’s somehow a criticism of me.”
In those circumstances, it’s difficult to know whether to approach the child directly or
the parent first. There are two schools of thought.
“I’d go to the child first,” says Andrew Fuller, author of Tricky ids. Usually a quiet
reminder that ’we don’t do that here’ is enough. ids nave finely tuned antennae (直觉) for
how to behave in different settings.”
He points out bringing it up with the parent first may make them feel neglectful, which
could cause problems. Of course, approaching the child first can bring its own headaches, too.
This is why White recommends that you approach the parents first. Raise your concerns with
the parents if they’re there and ask them to deal with it,” she says.
Asked how to approach a parent in this situation, psychologist Meredith Fuller
answers”Explain your needs as well as stressing the importance of the friendship. Preface your
remarks with something like ’I know you’ll think I’m silly but in my house I don’t want…’”
When it comes to situations where you’re caring for another child, white is straightforward
“common sense must prevail. If things don’t go well, then have a chat.”
There’re a couple of new grey areas. Physical punishment, once accepted from any adult,
is no longer appropriate. “A new set of considerations has come to the fore as part of the debate
about how we handle children.”
For Andrew Fuller, the child-centric nature of our society has affected everyone” The rules
are different now from when today’s parents were growing up,” he says, “Adults are scared
of saying ’don’t swear’, or asking a child to stand up on a bus. They’re worried that there
will be conflict if they point these things out – either from older children, or their parents.”
He sees it as a loss of the sense of common public good and public courtesy (礼貌), and
says that adults suffer form it as much as child.


Meredith Fuller agrees “A code of conduct is hard to create when you’re living in a world
in which everyone is exhausted from overwork and lack of sleep, and a world in which nice people
are perceived to finish last.”
“it’s about what I’m doing and what I need,” Andrew Fuller says. ”the days when a kid
came home from school and said, “I got into trouble”. And dad said, ‘you probably deserved
it’. Are over. Now the parents are charging up to the school to have a go at teachers.”
This jumping to our children’s defense is part of what fuels the “walking on eggshells”
feeling that surrounds our dealings with other people’s children. You know that if you
remonstrate(劝诫) with the child, you’re going to have to deal with the parent. it’s admirable
to be protective of our kids, but is it good?
“Children have to learn to negotiate the world on their own, within reasonable boundaries,”
White says. “I suspect that it’s only certain sectors of the population doing the running to
the school –better –educated parents are probably more likely to be too involved.”
White believes our notions of a more child-centred, it’s a way of talking about treating
our children like commodities(商品). We’re centred on them but in ways that reflect positively
on us. We treat them as objects whose appearance and achievements are something we can be proud
of, rather than serve the best interests of the children.”
One way over- worked, under-resourced parents show commitment to their children is to leap
to their defence. Back at the park, Bianchi’s intervention(干预) on her son’s behalf ended
in an undignified exchange of insulting words with the other boy’s mother.
As Bianchi approached the park bench where she’d been sitting, other mums came up to her
and congratulated her on taking a stand. “Apparently the boy had a longstanding reputation for
bad behaviour and his mum for even worse behaviour if he was challenged.”
Andrew Fuller doesn’t believe that we should be afraid of dealing with other people’s
kids. “look at kids that aren’t your own as a potential minefield,” he says. He recommends
that we don’t stay silent over inappropriate behaviour, particularly with regular visitors.
32. What did Stella Bianchi expect the young boy’s mother to do when she talked to him?
A. make an apology
B. come over to intervene
C. discipline her own boy
D. take her own boy away
33. Due to the child-centric nature of our society,
A. parents are worried when their kids swear at them


B. people think it improper to criticiids in public
C. people are reluctant to point our kids’ wrongdoings
D. many conflicts arise between parents and their kids
34. In a world where everyone is exhausted from over work and lack of sleep, .
A. it’s easy for people to become impatient
B. it’s difficult to create a code of conduct
C. it’s important to be friendly to everybody
D. it’s hard for people to admire each other
35. How did people use to respond when their kids got into trouble at school?
A. they’d question the teachers
B. they’d charge up to the school
C. they’d tell the kids to clam down
D. They’d put the blame on their kids
第二节(共5小题;每小题2分,满分10分)
根据短文内容,从短文后的选项中选出能填入空白处的最佳选项。选项中有两项为多余选项。
36 Shenzhou 1 lifted off from Jiuquan Satellite Launch Center on November 20, 1999.
Shenzhou 1 successfully landed in central Inner Mongolia after 21 hours in orbit, finishing its
first flight. This successful flight laid a solid foundation for Chinese manned Spaceflight.
Since then, Shenzhou 2—5 were successfully launched.
Shenzhou 6 was successfully launched at 900 a.m. on October 12, 2005. It carried two
astronauts for five days and safely returned to Earth at dawn on October 16, 2005, making China’s
manned space technology more mature. 37
Shenzhou 7 was launched on September 25, 2008, and successfully put three taikonauts(太
空人) into space. 38 That means China’s first step into space. During this mission, shenzhou
7 also sent a small satellite into space. Shenzhou 7 safely landed on earth on September 28,
which means the whole mission was completely successful. 39 It was also the third milestone
in the history of manned spaceflight in China.
Tiangong 1 lifted off from the Jiuquan Satellite Launch Center at 916 p.m. on September 30,
2011 as planned. 40 An unmanned spaceship will be launched in November to join together in
space with Tiangong 1. Two more missions are scheduled for next year and astronauts will board
Tiangong 1, which can also function as a space lab in the future space station. If the mission
succeeds, China will become the third country to master spacecraft rendezvous and docking(太
空站) technology following the then Soviet Union and the United States, experts said.


A. Gas and liquid are unable to mix on Earth, but in space they mix naturally.
B. It was an announcement to the world that China had grasped the key technology in
spacewalking.
C. The purpose of China’s manned space station is to peacefully explore space, and through
it, serve mankind.
D. China’s manned spaceflight program was officially approved in 1992.
E. One of them, two days later, made a monumental spacewalking.
F. The launch paves the way for China’s first space station mission.
G. It was the first time for China to have men engage in scientific experiments in space.
第一节 完形填空(共20题;每小题1.5分,满分30分)
阅读下面短文,从短文后各题所给的四个选项(A、B、C和D)中,选出可以填入空白处的最佳选项。
An adult giraffe's head is about six feet above its heart. This means that to 41 enough blood
up to the brain the circulatory 42 must be strong enough to keep the blood at very high pressure.
Biologists have known for some time that giraffes solve this problem by having 43 high blood
pressure, about 44 that of human beings. But an international team of biologists began to 45
about this. If giraffes have such high blood pressure, they should have a 46 problem with swelling
in their legs and feet. Why don't giraffes have swollen feet?
Giraffes should have 47 problem, too. Every time they bend heads 48 to drink, the blood
should 49 to their heads and have a hard time 50 back up (when the head is down) to the heart.
How come giraffes don't black out when they drink?
The answer to the 51 feet problem, the researchers found, is that giraffes have 52 the
researchers call a
legs and feet are 55 stiffer and tougher than those of other 56 . As a result, the blood vessels
in the leg cannot swell.
Therefore, the blood has nowhere to go but back to the heart. What about blood rushing to
the head 57 the giraffe bends down to drink? The researchers found that the giraffe's jugular
vein, which 58 blood from the head back to the heart, has lots of one-way valves in it. In the
giraffe's neck, there are lots of muscles that flex and relax repeatedly as the animal moves
its head and sucks 59 drinking water. By squeezing the valved jugular vein, they 60 blood moving
back to the heart even while the animal is drinking.
41. A. bring B. produce
B. system
C. transfer
C. function




D. pump
D. organism
D. commonly
42. A. structure
43. A. unusually B. generally C. uncomfortably


44. A. half B. multiple
B. wonder
C. double D. pair
45. A. investigate
46. A. terrible
47. A. other
48. A. up
49. A. crush




C. undertake D. learn
D. advisable B. unreliable
B. some


C. unsolvable
C. others

D. another
B. down

C. toward D. aside


D. rush B. brush
B. returning
C. push
50. A. following
51. A. healthy
52. A. what
C. plowing


D. pouring
B. swollen

C. dreary
C. that
D. radical


D. those
D. turns
D. organs
D. less
B. where
53. A. reveals
54. A. tissues
55. A. many


B. indicates
B. vessels
B. very
C. figures
C. pores
C. much




56. A. giraffes
57. A. whenever
58. A. reflects
59. A. in
B. animals
B. whatever
C. people


D. creatures


D. wherever
D. carries
C. however
C. receives

B. releases
B. up C. to D. from
D. keep 60. A. permit B. prevent C. retain
第II卷

第二节(共10小题;每小题1.5分,满分15分)
阅读下面材料,在空白处填入适当的内容(1个单词)或括号内单词的正确形式。
With the ___61___ (develop) of industry, air pollution is getting more and more serious.
In Beijing, many people 62 (suffering) from different kinds of 63 ( ill ) because
of air pollution.
Air pollution is caused 64 the following reasons about half of the problem is caused
by vehicles(车辆). There are more and more cars, buses on the roads, and they give off 65
(poison) gases. 25% of air pollution is caused by factories. Another 66 (fact) is the
smokers. Smoking not only does harm to their health but also to others’. 67 these, about
10% of air pollution is caused by other reasons.
We should take some measures to fight 68 pollution. New fuel can be used to take the
place of gas. We can plant more trees. If everybody realizes the importance of 69
(environment) protection and does something to stop pollution, the problem will 70 (solve).
第四部分 写作(共两节,满分35分)
第一节 短文改错(共10小题;每小题1分,满分10分)


增加:在缺词处加一个漏字符号(∧),并在下面写出修改后的词。
删除:把多余的词用斜线()划掉。
修改:在错的词下画一横线,并在该词下面写出修改后的词。
注意:1. 每处错误及修改均仅限一词。
2. 只允许修改10处,多者(从第11处起)不计分。
My parents and I went to the park on last Sunday. There were lots of visitors stood in front
of the ticket window. We waited a

long time and buy three tickets. In the Tiger Mountain of the park, I was too eager to see the
fierce frightened animals that I

quickened my steps through the crowd. Unfortunate, I got separated from my parents. I had hard time
looking for him, but I had no

ing in the park, I felt alone without any companions. Worse still, I had
no money, so I had to walk

home,covered as much as 5 kilometers.
第二节 书面表达(满分25分)
Dear editor,
I’m a Senior Three student. I’m unhappy these days. Last Tuesday, I was doing my
homework with my favorite MP3 on in Self-Study Class. Suddenly my head teacher took my MP3 away
from behind and the next day he scolded me before the whole class. I reasoned with him, only
to make things worse. Is there anything wrong enjoying music? What can I do?

Yours sincerely,

Li Hua
注意:回信中应包括以下要点:(可根据内容要点适当增加细节,以使行文连贯)
1. 你也是音乐爱好者
2. 音乐的益处
3. 音乐和学习关系的处理
4. 老师的意图


5. 和老师沟通





































参考答案与解析
A篇阅读
【答案】21.C
22.B
23.B
24.A
B篇阅读
25.【答案】C
【解析】根据题干关键词grandparents in their seventies with a 10-year-old boy找到文章对应Full
Price Total Ticket部分的Senior(age 65+):$$34.95 ,Child(ages 3-12):$$32.95和Total Ticket Online
Discount 中的Child(ages 3-12):$$29.95,Senior(age 65+):$$31.95进行计算(34.95x2+32 .95)-
(29.95+31.95x2)=9可知在上购票可省$$9,故选C。
26.【答案】B
【解析】根据题干关键词AT&T Dolphin Tales找到文章对应段是AT&T Dolphin Tales部分的The gallery
and viewing window are open to general admission guests periodically throughout the day排除
A;根据该部分的Don’t forget-AT&T Dolphin Tales is included in Total Ticket general admission
排除C和D;根据该部分的so don’t miss this spectacular show可推断海豚表演值得看看。故选B。
27.【答案】C
【解析】根据题干关键词With a Total Ticket找到文章对应部分4D Theater—Deepo’s Undersea 3D
Wondershow Presented by AT&T的Admission to the 4D Theater is included in Total Ticket general
admission!可有有全票可以游览4D Theater-Deepo’s Undersea 3D Wondershow。不需要额外付费。故
选C。
C篇阅读
28. 【答案】C.
【解析】该题比较难定位。由题干中的misjudgment一词可以勉强将答案锁 定在第三段第二句。该题主要
是考查对题干中短语stemmed from(源自于)的辨认和理解, 还考察Tiffany错误判断的原因,而选项中
的ignorance(无知)与misunders tanding意义相近,因此C正确.
29. 【答案】B.


【解析】由题干Mindsets LLC定位到第一个小标题下第一段最后两句。根据 原文中training一词不难判
断,该机构是一个培训机构,选项B与此相符。
30. 【答案】D.
【解析】由题干中的Dale定位到第二个小标题下第一段最后三句。该小标题的主要 内容是在说明隐性别
的原因造成对他人认识的偏差。文章以Dale为例说明参加这种学习班的作用,并 且从后面一段中Dale的
亲身经历来说明,参加该班的作用对Dale来说就是消除了他在性别上的偏 见。选项D与此相符。
31. 【答案】C.
【解析】因为第二个小标题下通篇都在讲D ale和workshop,答案没有明确的出处,这时可以看该标题下
的首段与末段,通常会讲到陈述 人的观点。该题是在考查Dale对Mindsets LLC workshop的认识。根据
第二个标题下最后一段末句可以确定答案为C.
D篇阅读
32.【答案】A
【解析】由提干中的Stella Bianchi, mother 定位到第三段1-2句:“The boy’s mother ran toward
me from across the park ,” I thought she was coming over to apologize, but instead she started
shouting at me for ‘disciplining her child ’.文章提到Stella本以为孩子的母亲是来道歉的,A
与此一致。本体关键在于定位答案。
33.【答案】C
【解析】由提干中的child-centric nature 定位到第十六段3—4句:“Adults are scared of saying,”
‘don’t ware’, or asking a child to stand up on a bus. They’re worried that there will be
conflict if they point these things out —either from older children, or their parents.’’文章提到现在父母害怕直接告诉孩子不要骂人或让孩子们在公交车上站起来,因为他们担心如果这样会引起矛盾,可知人们不愿指出孩子的缺点。C与此相符。
34.【答案】B
【解析】由题中的overwork和lack of sleep定位到第十八段第2句:A code of conduct is hard to create
when you’re living in a word in which everyone is exhausted from overwork and lack of sleep,
and a world in which nice people are perceived to finish last.该段第二句与B几乎完全一致,只
是做了语序调整,本题关键在于定位答案。
35.【答案】D
【解析】由提干中的trouble和at school定位到第十九段第2句:“The days when a kid came from school
and said, ‘I got into trouble’, and dad said, ‘you probably deserved it’, are over.文章

< p>
提到当孩子告诉父母遇到麻烦时,父亲会说“或许你是活该”,可见家长责备孩子。本题关键在于看 清题
干询问过去家长的做法。
七选五
【答案】36.D
37.G
38.E
39.B
40.F
完型填空
41. D 词义辨析 题。文章第一句就指出,成年长颈鹿的脑袋在心脏以上6英尺高的地方。空格所在句子意
思是:这意味着 ,要________足够的血液到大脑,循环________必须足够强壮才能保持血液有很强的压力。pump除了表示“用泵抽”之意,还可以表示“(用泵拉取)注入”,符合句意,所以D正确。
42. B 词义辨析题。由空格所在句意可知,空格处是指长颈鹿的循环系统,所以B正确。
43. A 词义辨析题。空格所在句意为:生物学家已经发现长颈鹿有________很高的血压能 解决这个问题,
由此可知,这个很高的血压应该是与众不同的,所以A正确。
44. C 词 义辨析题。根据空格前面句子意思可知,空格处应该是一个表示倍数的词。double名词表示“两
们 ”,符合句意,所以选C。前面提到长颈鹿的血压很高,所以排除A;multiple只是指“倍数,若干”,
但是没有说明是多少倍,所以排除。
45. B 词义辨析题。空格所在句子句首的but表 示转折,说明句子内容与前面提到的内容不同,选项中只
有wonder一词可以表达这种不同,即“一 个国际生物学家小组对此表示怀疑”,所以B正确。
46. A 词义辨析题。空格所在的句意为:如 果长颈鹿的血压有这么高,那么它们会有________的问题。选
项中terrible意为“很糟 糕的”,unreliable意为“不可靠的”,unsolvable意为“不能解决的”,
adv isable意为“可取的”,只有terrible符合句意,所以A正确。
47. D 词义辨析 题。通读第三段可知,该段是在讲长颈鹿血压很高的话为什么喝水时不会晕倒这个问题,
所以该空格所在 句子意思是“长颈鹿还应该有另一个问题”,而不是“有其他问题”,所以D正确。排除
A。
48. B 词义辨析题。根据常识可知,长颈鹿要把头低下来喝水,所以B正确。
49. D 词义辨析题。本句意思是长颈鹿低头喝水时,血液应该________到大脑。rush意为“冲;急流” ,
符合句意,所以D正确。crush意为“压碎”,brush意为“刷”,push意为“推动”。
50. C 词义辨析题。根据句意可知,长颈鹿头低下的时候,血液流回到心脏比较困难,所以C正确 。return
作不及物动词时虽然也可以表示“返回”,但它本身已经包含back的含义,所以排除 。
51. B 词义辨析题。联系上下文可知,如果长颈鹿血压很高,那么它们会有腿部肿胀的问题, 空格所在句
子就是指长颈鹿对于腿部肿胀问题的解决,所以B正确。原文第二段已经出现swollen feet这一表达,


根据原词复现的原理也可知本题选B。
52. A 语法结构题。空格为所在句子从句的引导词,在从句中作宾语,所以A正确。
53. D 惯用搭配题。选项中能与out搭配的词有figure和turn, figure out表示“计算出;解决”,不
合句意,排除;turn out表示“证实;发觉是”,符合句意, 所以D正确。reveal意为“揭示”,indicate
意为“指出”。
54. A 词 义辨析题。空格前的and说明空格处的词和skin并列,句意为“它们腿和脚上的皮肤和其它
___ _____”。tissue意为“组织”,符合句意,所以A正确。vessel意为“血管”,pore意为 “小孔”,
organ意为“器官”。
55. C 语法结构题。空格后为两个形容词比较级 ,由此可知空格应该是能够修饰形容词比较级的词,结合
句意可知长劲鹿腿和脚的皮肤和其他组织更坚硬 ,所以much符合句意,C正确。
56. B 词义辨析题。由空格前的than可知,是拿长劲鹿和其他动物做比较,所以B正确。
57. A 逻 辑衔接题。空格所在句子意思是:________长劲鹿低头喝水时,血液冲向大脑怎么办?根据句意
可知whenever符合,所以A正确。
58. D 词义辨析题。空格所在句子意思为:(长劲鹿 的劲静脉)能将血液从大脑________回心脏。carry符
合句意,所以D正确。
59. B 惯用搭配题。suck up意为“吸收”,suck in意为“利用;欺骗;诈取”,to和from不能和suck
搭配,suck up符合句意,所以B正确。
60. D 词义辨析题。空格所在句意为:通过挤压劲静脉的阀门,它 们能________血液流回到心脏,甚至在
长颈鹿喝水的时候也能。keep符合句意,所以D正确 。
语法填空
【答案】61. development 62. suffer 63. illnesses 64. by 65.
poisonous
66. factor 67. Besides 68. against 69. environmental 70. be
solved
短文改错
My parents and I went to the park on last Sunday. There were lots of visitors stood in
front of the ticket window. We waited


standing
a long time and buy three tickets. In the Tiger Mountain of the park, I was too eager
to see the fierce frightened animals that I


bought
so frightening
quickened my steps through the crowd. Unfortunate, I got separated from my parents. I
had ∧ hard time looking for him, but I
Unfortunately
a
them
had no luck. Wandering in the park, I felt alone without any companions. Worse still, I had
no money, so I had to walk home,

lonely
covered as much as 5 kilometers.
covering
书面表达
One possible version
Dear Li Hua,
I’m sorry that you have trouble with your head teacher. From your letter I know you
are a music fan. So it is with me. As every one knows, music can be a good companion, which can
bring you happiness, forgetting worries at school. One can’t live without music. No wonder you
have a great passion for music. If you enjoy music in a proper time as well as in a proper way,
you can keep on studying in a better state. I think you should understand your teacher’s intention.
He just cared too much about you. You’d better have a heart-to-heart talk with him so that you
can understand each other much better. I’m sure music is the bridge to the soul.
Good luck!
Tom


高考模拟英语试卷
第一部分 听力(共两节,满分30分)
第一节(共5小题;每小题l 5分,满分7.5分)
听下面5段对话。每段对话后有一个小题,从题中所给的A、B、C三个选项中选出最佳选项,并标
在试 卷的相应位置。听完每段对话后,你都有l0秒钟的时间来回答有关小题和阅读下一小题。每段对话
仅读 一遍。
1. Where is Chow’s Chinese Food?
A. Behind FC. B. On Walnut Street. C. Opposite Mort’s Gym.
2. Why did the man leave the company?



A. He didn’t like the office manager.
B. He was dissatisfied with the salary.
C. The working environment was not enjoyable.
3. How will the man get to work today?
A. By bus. B. By car. C. On foot.
4. Where does the conversation take place?
A. In a restaurant. B. In a supermarket. C. In the woman’s house.
5. What are the speakers mainly talking about?
A. An opera. B. A singer. C. A design.
第二节 (共15小题;每小题1 5分,满分22 .5分)
听下面5段对话或独 白。每段对话或独白后有几个小题,从题中所给的A、B、C三个选项中选出最
佳选项,并标在试卷的相 应位置。听每段对话或独白前,你将有时间阅读各个小题,每小题5秒钟;听完
后,各小题给出5秒钟的 作答时间。每段对话或独白读两遍。
听第6段材料,回答第6、7题。
6. What is the relationship between the speakers?
A. Husband and wife. B. Workmates. C. Neighbours.
7. What is the man most worried about?
A. The frequent noise. B. The air pollution. C. The traffic jam.
听第7段材料,回答第8、9题。
8. Where are the speakers?
A. In a shopping center. B. On a bus. C. In a hospital.
9. When will the woman get to Santa Anita shopping center?
A. In about 15 minutes.


B. In about 20 minutes.
C. In about 35 minutes.
听第8段材料,回答第10至l2题。
10. What does the man want to buy?
A. Beds. B. Chairs. C. Tables.
11. Why doesn’t the man like the white one?
A. Its colour is not suitable. B. It’s very expensive.
12. How does the man pay?
A. In cash. B. By credit card. C. By cheque.
C. It’s uncomfortable.
听第9段材料,回答第13至l6题。
13. What is the man’s main task in the early morning?
A. To check the buildings.
B. To give instructions to all employees.
C. To wait for his manager.
14. What is the man’s most important duty during the day?
A. To check the lights and bathrooms.
B. To ensure the safety of visitors.
C. To make sure the valuable things are kept safe.
15. What is necessary to work in the castle?
A. Being physically fit.
B. Being an expert in history.
C. Being a retired police officer.
16. What can we learn from the conversation?
A. The staff in the castle are quite responsible.
B. Visitors are admitted to the castle after 730.
C. There are lifts in the castle.
听第l0段材料,回答第17至20题。
17. Why did the speaker take the sailing to the North Pole?
A. He was inspired by some friends.
B. He was crazy about adventure.
C. He was encouraged by a book.
18. What is the biggest difference between sailing on the Arctic seas and normal seas?
A. The temperature is changeable there.


B. One should have three layers of clothing.
C. Help can’t be expected in case of danger.
19. What happened to the speaker on the way to the North Pole?
A. He was attacked by a polar bear.
B. He caught sight of whales.
C. His boat broke down a few times.
20. What do we know about the speaker?
A. He finds his trip rewarding.
B. He felt it really hard to sail on the Arctic Seas.
C. He can only get to certain unrestricted areas.

第一节 单项填空(共15小题;每小题1分,满分15分)
21. The education department ________married women to return to teaching to fill the gaps for both
part-time and full-time teachers.
A. adjusted to B. referred to C. applied to D. appealed to
22. A lack of ________ of cultural differences is likely to create problems.
A. existence B. revolution C. awareness D. evidence
23. He was a non-smoker who kept himself fit and was ________about what he ate.
A. familiar B. popular C. regular D. particular
24. I appreciated ________the opportunity to work with the foreign expert two years ago.


A. to have given
C. having given










B. to have been given
D. having been given
25. –When shall I go to the meeting?
– ________. You may go right now or leave for it in a few minutes.


A. It’s hard to say





B. It’s up to you
D. It’s all right C. It’s a waste of time
26. Generally speaking, if one grows up in ________large family, he or she is more likely to develop
________ability to get on well with others.
A. a; an B. the C. the; an D. a; the
27. – Did you finally reach the top of the mountain?
– Yes. Even I myself didn’t believe that I could make ________.
A. one B. it C. that D. this
28. I ________ to make him a phone call to congratulate him on his success, but I ________it.




A. hoped, hadn’t managed
C. hope, don’t manage




B. has hoped; didn’t manage
D. had hoped; didn’t manage
29. Gun control is a subject ________Americans have argued for a long time.
A. of which B. with which C. into which D. about which
30. ________her mother after being separated for 20 years, she couldn’t help, even though she tried not to,
________.
A. Seen; crying B. To see; cry C. Seeing; crying D. Having seen; cry
31. I just don’t understand ________that leads to so many teenagers becoming addicted to playing
computer games.
A. why it does B. what it does C. what it is D. why it is
32. There must have been a large member of tourists in Beijing during the National Day holiday, ________?
A. mustn’t there B. haven’t there C. didn’t there D. weren’t there
33. He got a call from his mother, or he ________doing his assignment ten minutes ago.


A. had finished
C. could finish












B. would have finished
D. finished
34. Never before ________planted so many trees and devoted so much to pollution treatment.
A. had we B. we had C. have we D. we have
35. The parents are making some changes to their previous education methods, of ________ negative effects
they begin to be aware.


第二节 完形填空 (共20小题;每小题1.5分,满分30分)
Today children always want to get anything they want at home, but they wouldn’t do housework. To
teach my children a 36 , I decided to go on a cleaning strike for a(n) 37 , but I wouldn’t
announce it.
38 telling my girls, I did no picking up or cleaning those days. I still cooked and washed clothes,
but things that they could 39 do themselves, such as tidying up after themselves, I wouldn’t do. Would
a week be enough for Lois and Joe to notice their mess and 40 their tidy gene? I started 41 to
see the result.
Day One of the strike passed as 42 . Lois and Joe left for 43 at 8 am, leaving behind bowls
and dishes on the table. With great efforts, I ignored the 44 and headed out to work directly. Of
course, the children’s bowls were still there when they came back from school. Where was their 45
point? I desired to clear the mess, but it didn’t seem to 46 them. In fact, I didn’t think they even
A. who B. whom C. whose D. which


noticed.
Day Two and Three followed the 47 pattern, and both the coffee table and the kitchen worktop
were filled with some pretty dirty dishes. I 48 whether I had the staying power to see this through. By
Day Four, however, I saw some signs of 49 and they developed a taste for order. When I opened the
door, I was 50 to see everything in the kitchen was where it should be. After four days, my children
were so sick of the mess, so they cleaned up themselves. Since then, they have kept the 51 of helping
me with the housework. They 52 to wash the bowls while I tidied the room. 53 , every day they
put their bedrooms in order.
I thought my experiment was 54 . As long as parents had 55 with children, they could
learn many things from everyday life.
36. A. skill
37. A. week
38. A. Without
39. A. regularly
40. A. appreciate
41. A. suggesting
42. A. well
43. A. cinema
44. A. food
45. A. laughing
46. A. frighten
47. A. traditional
48. A. doubted
49. A. progress
50. A. proud
51. A. task
52. A. determined
53. A. What’s more
54. A. cruel
55. A. quarrel

第三部分 阅读理解(共20小题;每小题2分,满分40分)
阅读下列短文,从每题所给的四个选项(A、B、C、和D)中,选出最佳选项。
B. custom
B. day
B. By
B. easily
B. abandon
B. refusing
B. always
B. hospital
B. dishes
B. crying
B. bother
B. humorous
B. believed
B. confidence
B. curious
B. habit
B. pretended
B. After all
B. negative
B. patience
C. lesson
C. hour
C. After
C. hardly
C. improve
C. planning
C. follows
C. school
C. mess
C. succeeding
C. threaten
C. different
C. hoped
C. responsibility
C. amazed
C. demand
C. planned
C. In short
C. beneficial
C. fight
D. regulation
D. month
D. Besides
D. eagerly
D. awaken
D. expecting
D. regards
D. office
D. disaster
D. breaking
D. embarrass
D. same
D. imagined
D. disappointment
D. disgusted
D. wish
D. volunteered
D. Worse still
D. worthless
D. deal


A
A British scientist whose schoolmasters told him he was too stupid to study science has been awarded
the Nobel Prize in medicine for his pioneering work on cloning.
At the age of 15, John Gurdon ranked last out of the 250 boys in his school at biology, and was in the
bottom set in every other science subject. Sixty-four years later he has been recognized as one of the finest
minds of his generation after being awarded the £750,000 prizes, which he shares with Japanese stem cell
(干细胞)researcher Shinya Yamanaka.
John revealed that his school report, which noted that for him to study science at University would be
a waste of time, still sits on his desk at the Gurdon Institute in Cambridge. After receiving the report he
switched his attention to classics and was offered a place to study at Christ Church, Oxford, but was
allowed to switch courses and read zoology instead.
It was at Oxford as a postgraduate student that he published his research on genetics and proved for
the first time that every cell in the body contains the same genes. He did so by taking a cell from an adult
frog’s intestine (肠), removing its genes and putting them into an egg cell, which grew into a clone of the
adult frog.
The idea was controversial at the time because it contradicted previous studies, and it was a decade
before his work became widely accepted. But it later led directly to the cloning of Dolly the Sheep by Prof
Ian Wilmut in 1996.
He received a call from the Swedish Academy telling him he had won the prize, but he was initially
suspicious that the call was a trick by a friend or colleague speaking in a Swedish accent.
56. When John Gurdon was fifteen, he________.




A. performed badly in science subjects.
B. ranked last in every subject in his school
C. showed great interest in working on cloning
D. was not clever enough to go on with his study
57. According to the passage, the school report________.




A. meant nothing to John Gurdon
B. had a long-term effect on John Gurdon
C. encouraged John Gurdon to work harder on science
D. advised John Gurdon to study classics at Christ Church
58. How did John Gurdon prove that his idea is right?
A. By reasoning.







B. By questioning.
C. By doing experiments D. By quoting(引用) others’ research


59. When John Gurdon was informed of his winning the Nobel Prize, he felt________.
A. excited B. calm C. relaxed

B
To help you get more for your money, we’ve teamed up with HiF to offer highly competitive
exchange rates to both individuals and businesses. It doesn’t charge any hidden fees.
Built from their experience of managing billions in foreign exchange each year, HiF has created a range
of simple international payment services designed to save your time and money.
For smaller amounts, HiF Online provides you with the quickest and easiest way to convert (兑换)
£50-£300,000 online at the touch of a button wherever you have access to the Internet and as it’s all
online, there’s no need to stand in line at a foreign exchange counter.
For larger transfers (转移), their telephone based on the foreign exchange service gives you access to
your dealer who will get to know you and your individual requirements providing a high level of personal
service.
Finally, if you’re making regular payments, like salary and overseas pension transfers, school fees, the
HiF Regular Transfers service offers these in convenience.
It is used by 97 of the world’s top 100 banks, so you’re fully protected online and you can manage your
account and track payments 24 hours a day. It’s quick to sign up, free and you get all of these great benefits
Move money, pay people and settle bills.
Save time and money by sending your international money transfers the easy way.
Manage your account 247 whenever it suits you.
Get quotes, make transfers and track payments wherever you have access to the Internet.
Your security is our priority (优先考虑的事).
We work hard to keep your money safe and use the same security as 97 of the World’s 100 largest
banks and over 93% of 500 companies.
Choose how you pay.
To make your life easier, HiF accepts a wide variety of payment methods.
Help and support.
We understand that people who make international money transfers may seem a little uptight. But
you needn’t worry; the feeling that you feel nervous will disappear for we’re here to help.
60. HiF Online covers the following services ECEPT________.


A. exchanging money
C. doing shopping online








B. transferring money
D. transferring school fees
D. doubtful


61. Which of the following statements are the reasons why people are advised to use HiF Online?
① It offers highly competitive rates.
② It is fast but difficult to operate.
③ It is highly secure for customers.
④ It offers many payment methods.
A. ①②③ B. ②③④ C. ①②④ D. ①③④
62. The underlined word “uptight” in the last paragraph has the closest meaning to ______.
A. disappointed B. tense C. frightened D. excited
63. What is the author’s purpose of writing this passage?





C
More and more people are taking their iPads to bed with them to surf the web, check Facebook or send
emails before switching off the light. But researchers are warning that the blue light their screens give off
can stop users getting a good night’s sleep. That is because this type of light is similar to daylight,
convincing the brain that it is still daytime. Blue light prevents the production of a brain chemical called
melatonin (褪黑激素),which helps us fall asleep. By contrast, light which is more orange or red does not
prevent melatonin production, perhaps because our brains recognize it as a cue (暗示)that the day ends.
Scientists have known for years that staring at screens late in the evening can affect sleep – be they
television screens, computer screens or mobile phone screens. However, because mobiles and iPads are by
nature portable – not to say addictive – more people are taking them into the bedroom. Users also tend to
hold them much closer to their eyes than a computer or television screen.
Researchers are warning that looking at iPad displays for more than two hours leads to lower natural
melatonin levels as the devices give off blue light. They say, “Turning off the devices at night is the final
solution. But if you have to use these devices at night, dim the display to cut down irritation (刺激) of the
eyes and limit the time you spend on them before bed.”
They wrote in the journal Applied Ergonomics that iPad makers should adjust the spectral power
distribution (光谱功率分布) of the devices so that they affect the sleep patterns of users less.
It is not just a good night’s sleep that could be harmed by too much late night screen time. Researchers
know that long-term harm to sleep patterns can lead to an increased risk of obesity and even cancers.
A. To advertise for a special company.
B. To introduce an exchange service briefly.
C. To guide people how to transfer their money.
D. To make an advertisement for HiF Online.


64. Blue light prevents iPad users from sleeping well because ________.




A. it makes the brain more active
B. it can help decrease the production of melatonin
C. it is brighter than light that is more orange or red
D. it is the same as daylight which keeps people awake
65. The underlined phrase “dim the display” means ________.


A. turn off the display

B. keep away from the display
D. make the display less bright C. clean the display carefully
66. In the last paragraph, the author intends to ________.




A. show the danger of obesity and cancers
B. call on researchers to study sleep patterns
C. warn of the seriousness of long-term poor sleep
D. show how sleep is harmed by using iPads too much
67. What’s the main idea of the text?




A. People should limit the use of electronic devices.
B. Using iPads before bed can cause a poor night’s sleep.
C. Scientists are warning that blue light is bad for our health.
D. Looking at screens of iPads increases chances of sleeplessness.

D
Fishes are expected to be reduced in size by up to 24% because of global warming, say scientists.
Scientists believe that global warming has a greater effect on marine ecosystems (海洋生态系统) than
previously thought. Previous researches have suggested that increasing ocean temperatures would affect
both the distribution and the reproductive abilities of fishes. Now scientists believe that fish size would also
be heavily affected.
Researchers built a model to see how fishes would react to the lower level of oxygen in water. Ocean
temperatures increase and so do the body temperatures of fishes. But, according to the leading scientist Dr
William Cheung, the lower level of oxygen in water is the key. Warmer waters could reduce the ocean
oxygen level and greatly reduce fishes’ body weight. On the other hand, rising temperature directly increase
the metabolic (新陈代谢的) rate of fishes’ body function. This leads to an increase in oxygen demand for
normal body activities. So fishes will run out of oxygen for growth at a smaller body size.
The research group also used its model to predict fishes’ movements as a result of warming waters.
The group believes that most fishes’ populations will move towards the Earth’s poles at a rate of up to 3.3


km per year.
Taking these factors into consideration, the research team concludes that fishes’ body size will be
reduced by between 14% and 24%, with the largest reduction in the Indian and Atlantic Oceans.
When compared with the actual study of fish size, the model seems to make a lower judgment of
what’s actually happening in the seas. The researchers made actual studies on some kinds of fishes living in
the Atlantic Ocean. They found that the recorded data on these fishes showed greater reduction in body size
than the models had predicted.
Scientists believe smaller-sized fishes could seriously affect the ability of fishes to reproduce (繁殖). Dr
Alan Baudron, from the University of Aberdeen, the U, thinks that smaller fishes produce fewer and
smaller eggs and it could affect the reproductive potential of fishes.
68. From Paragraph 2 we can know that ________.




A. scientists have made two researches on the influence of global warming by now.
B. researchers suggest that the ocean temperature is increasing at a high rate.
C. researches suggest that global warming would change fishes’ inner structure
D. scientists have known three aspects of the influence that global warming has on fishes
69. Which is the most important factor that makes fishes smaller?




A. Higher body temperatures of fishes.
B. Lower oxygen level in water.
C. Slower heart beats of fishes.
D. Higher pressure in water.
70. According to the group, in 30 years, most fish populations will move about ________.
A. 30 kilometers
C. 100 kilometers










B. 60 kilometers
D. 200 kilometers
71. Which of the following in the best title for the passage?




A. Climate change may make fishes smaller.
B. Global warming has a bad effect on fishes.
C. Global warming affects the distribution of fishes.
D. Climate change may affect the ability of fishes to reproduce.

E
An American program called the President’s Emergency Plan for AIDS Relief (PEPFAR) helps
millions of people infected with HIV, the virus responsible for AIDS. PEPFAR is considered one of the most
successful programs created during the presidency of George W. Bush. Its goal is to make HIVAIDS


change from a death sentence to a disease that, while serious, could be treated.
And for millions of people around the world, that is just what has happened. But researchers say it
was not until PEPFAR started using generic drugs (非专利药) that major progress was made in fighting
HIVAIDS. Generic drugs are copies of medicines developed by large drug companies. But they often cost a
lot less.
artik Venkatesh was the lead author of a report on the study. He says the high cost of patented drugs
had an immediate influence on the program after it began. American officials considered whether to
provide patented drugs for HIV- infected patients, both in the United States and overseas. Dr. Venkatesh
says some people believe that the drug industry wanted the government-financed program to use patented
drugs.
But generic drugs were shown to be effective in treating HIV as far back as 2003. Using generic drugs
helped cut the cost of treating a person from about 1,100 dollars a year to about 300 dollars a year.
PEPFAR has also been able to save millions of dollars by reducing transportation costs – for example,
using ships instead of airplanes to move the drugs. Dr. Venkatesh says, “The PEPFAR model could be used
in the fight against other diseases as developing countries begin seeing health problems that until recently
had only been seen in richer countries. Maybe the real and perhaps potentially larger burden in the future
is going to the chronic diseases (慢性病) like heart disease and cancer. And those are also diseases that
require long-term treatment. A lot can be learned form the experience in accessing generic drugs for the
treatment for HIV.”
72. Why was PEPFAR established?




A. To make it possible to cure AIDS.
B. To study how people are infected with HIV.
C. To warn people that AIDS is serious.
D. To help people develop patented drugs.
73. According to Dr. Venkatesh, generic drugs ________.




A. must be used with the permission of drug companies
B. contribute more to curing AIDS than patented drugs
C. are provided for people with AIDS for free
D. are copies of cheap medicines
74. Which of the following is TRUE about generic drugs?



A. Generic drugs are much easier to transport.
B. Generic drugs were created as far back as 2003.
C. Generic drugs don’t belong to the government-financed program.


D. Generic drugs can reduce the economic pressure of AIDS patients.
75. From the passage we can know that________.
A. HIV is considered as a burden larger than any other disease
B. new health problems will first be seen in developed countries
C. the PEPFAR model offers lessons for people to treat other diseases
D. people ought to understand more about fighting long-term diseases



第 II 卷
第四部分 写作(共两节,满分35分)
第一节 短文填词(共10小题;每小题1分,满分10分)
Two little children were sitting by the fire one cold winter night.
Suddenly they heard a k________ at the door, and one ran to open it.

_
_
_
There, o________stood a child with no shoes on his feet. He asked to
_
_
_
_
_
come in and warm________.They agreed. They drew the little
_
_
_
_
stranger to
_
_
_

their warm seat and shared their supper with him, to________they


gave



their bed, while they slept ________a hard bench. In the nigh they


_
_
_
were
_
_
_
_
_
w________up by sweet music. The stranger child stood before them
_
_
_
_
and
_
_

said, “I am the Christ Child, ________(漫步) through the world to



bring

_
_
_
_
peace and ________(幸福) to all good children. I’ll give you a gift.”
_
_
_
_
_
He
_
_
_
_
broke a branch from the tree and p________ it in the ground. The
_
_


branch
grew into a great tree, and every year it bore wonderful ________(金



色的)





fruit for the kind children.















76. knock
77. outside
78. himself
79. whom
80. on
81. woken
82. wandering
83. happiness
84. planted
85. golden


第四节 书面表达(满分25分)
国庆长假引发了人们对于旅游景点该不该涨价的热议。上周你们班对此进行了 一次热烈的讨论。请
根据下面的讨论结果完成短文。
大约90%认为不该涨价
1. 旅游景点是公共资源;
2. 更多的人可以享受美景;
3. 涨价会增加人们的经济负担。
你的观点:……

要求:1. 词数120左右,可以适当增加细节,以使行文连贯;
2. 词汇:景点scenery spot; 维护maintenance。
注意:开头已给出,不计入总词数。



Last week my class had a heated discussion about whether prices of scenery spots should be raised.
The students held different opinions about it. ______________________________
______________________________________________ _____________________________________________
_ __________________________________________________ ________________________________________
______ __________________________________________________ ___________________________________
___________ __________________________________________________ ______________________________
________________ __________________________________________________ _________________________
_____________________ __________________________________________________ ____________________
__________________________ __________________________________________________ _______________
_______________________________ __________________________________________________ __________
____________________________________ __________________________________________________ _____
_________________________________________ __________________________________________________
______________________________________________ _____________________________________________
_ __________________________________________________ ________________________________________
______ __________________________________________________ ___________________________________
___________ _______________________________________________



10%认为可以适当涨价
1. 景点建设及维护费用增加;
2. 避免超负荷的游客量对景点造成破坏


听力:CABCA; CBBCB; ACBBA; ACCBA
单选:DCDDB; DBDDC; CDBCC
完形:CAABD; DBCCD; BDAAC; BDACB
阅读:ABCD; CDBB; BDCB; DBCA; ABDC
短文填词
Two little children were sitting by the fire one cold winter night.
Suddenly they heard a k________ at the door, and one ran to open it.
There, o________stood a child with no shoes on his feet. He asked to
come in and warm________.They agreed. They drew the little stranger to
their warm seat and shared their supper with him, to________they gave
their bed, while they slept ________a hard bench. In the nigh they were
w________up by sweet music. The stranger child stood before them and
said, “I am the Christ Child, ________(漫步) through the world to bring
peace and ________(幸福) to all good children. I’ll give you a gift.” He

76. knock
77. outside
78. himself
79. whom
80. on
81. woken
82. wandering
83. happiness
broke a branch from the tree and p________ it in the ground. The branch 84. planted
grew into a great tree, and every year it bore wonderful ________(金色的) 85. golden
fruit for the kind children.
书面表达
Last week my class had a heated discussion about whether prices of scenery spots should be raised.
The students held different opinions about it.
About 90 percent of the students think it is not right to raise the prices of some scenery spots. They
think those places are public resources, which should be shared by more people. If the prices go up too
much, people will have to pay more money. Maybe some people can’t afford it.
However, 10 percent of the students think that it is acceptable to raise the prices a little. For one thing,
construction and maintenance of scenery sports need more money. For another, raising prices can be an
effective way of avoiding too many tourists rushing there in peak time. That may cause some damage to
scenery spots.
From my point of view, there is a balanced way that can be considered – raise prices during holidays
and lower prices in other time.


高考模拟英语试卷
考试时间:120分钟 试卷满分:150分

第一部分 听力(共两节,满分30分)
第三节(共5小题,每小题1.5分,满分7.5分)
听下面5段对话。每段对话后有一个小 题,从题中所给的A、B、C三个选项中选出最佳选项,并标
在试卷的相应位置。听完每段对话后,你都 有10秒钟的时间来回答有关小题和阅读下一小题,每段对话
仅读一遍。
1. What can we learn about the man’s shoes?
A. Smaller than his brother’s.
C. The same size as his brother’s.
2. What is Mary’s job now?
A. She is a secretary.
3. What time is it now?
A. Eight-thirty. B. Nine o’clock. C. Eight o’clock.
B. She is a reporter. C. She is a novelist.
B. Larger than his brother’s.
4. Where are the man and the woman at this moment?
A. Watching a movie. B. In line at a museum.
C. In line outside a movie house.
5. Why is the shirt nice?
A. It looks good. B. It matches the girl’s scarf.
C. It not only looks good, but also matches the girl’s scarf.
第四节(共15小题,每小题1.5分,满分22.5分)
听下面5段对话或独白。每段对话或独白后有几个小题,从题中所给的A、B、C三个选项中选出最佳选项,并标在试卷的相应位置。听每段对话或独白前,你将有时间阅读各个小题,每小题5秒钟;听完后,各小题将给出5秒钟的作答时间。每段对话或独白读两遍。
听第6段材料,回答第6-7题。
6. Why doesn’t the man want to go for a walk?
A. Because he is very tired. B. Because he wants to watch TV.
C. Because he has too much work to do.
7. What does the man hate doing?
A. Taking care of the child.
C. Going for a walk.
B. Going shopping.


听第7段材料,回答第8-10题。
8. Why did Mark call Jency?
A. To ask if her test has been well prepared.
B. To invite her out for an evening.
C. To offer his help with her test.
9. What will Jency do next Saturday afternoon?[]
A. Prepare her test.
B. Do nothing, and she will be free that afternoon.
C. Take her test.
10. What has Jency agreed to do with Mark next Saturday?
A. To see a film. B. To attend a concert. C. To have dinner.]
basketball game. B. About a test.
C. About books.
15. What does the man finally decide to do tonight?
A. Go to the basketball game. B. Go to a restaurant.
C. Prepare for the test.
16. What can we know from the dialogue?
A. The woman is a teacher of the man.
B. The man has been well preparing for the test.
C. The two speakers probably are classmates.
听第10段材料,回答第17-20题。
17. What was the man doing?
A. He was playing a trick. B. He was playing the violin for fun.
C. He was playing the violin on the street to earn money.

听第9段材
料,回答第
14-16题。
14. What are
the two
speakers
talking about?
A. About a



18. Where did the man stand when he played?
A. In the middle of the pavement.
C. On a bridge near the station.
19. How did the man feel when he finished his first tune?
A. He felt depressed.
C. He felt excited.
20. Which one is NOT true according to the man?
A. He played the violin rather bad so that nobody listened to him.
B. He thought he would be arrested or told to shut up at first.
C. He would continue to have this style of living.

第二部分 阅读理解(共两节,满分40分)
第二节(共15小题;每小题2分,满分30分)
阅读下列短文,从每题所给的四个选项(A、B、C和D)中选出最佳选项,并在答题卡上将该项涂
黑。
A
I’ve often wondered if I might do more good as a travel agent rather than as a psychologist.
It seems that I have been more dramatically affected by certain kinds of travel experiences than
I ever have.
My trip to Iceland is a fine example of that. The plan was to spend two days in a remote mountain
hut in Iceland. I was working on a photographic book about winter in Iceland and needed to capture
images of this amazing region of high mountain peaks, smoky volcanoes, and lakes with floating
icebergs.
The moment after we arrived, the weather turned extreme making visibility impossible. It snowed
so much and the wind blew so hard that we couldn’t leave the tiny hut. To stay warm, we walked
around in circles much of the day inside the tiny hut. We tried to call for help but the radio
did not work. Day after day, we watched our supplies of food and fuel grow dangerously short.
We got acute cabin fever (幽居病) and started going for walks and ski expeditions outside. Even
when the weather finally broke, nobody came to get us even though it was three days beyond our
scheduled pickup. By the time the rescue team came to pull us out, we had all given up hope.
From then on, the world looks different to me, as does my life. It would have taken me years
of psychotherapy to get to the same point.
B. He felt ashamed of himself.
B. Under a bridge near the station.


Almost everyone has a story to tell, and interestingly, most of these experiences were not
altogether pleasant at the time. In fact, it appears that the most constructive life-changing
journeys were those that involved some sorts of awful and uncomfortable events that forced the
person to develop new resources, increase confidence, and solve problems in new ways.
21. The writer went to Iceland to __________.
A. enjoy the natural beautiful floating icebergs
B. collect materials for psychological research
C. take photos about the region for a book
D. challenge the high mountains there
22. During the trip in Iceland, those travelers faced difficulties ECEPT that __________.
A. they were short of food and fuel
B. they got lost in the mountain
C. they couldn’t see the surroundings clearly
D. they failed to get in touch with the rescue team
23. From Paragraph 3, we learn that the travelers __________.
A. stayed in the hut for three days altogether
B. were in despair before they were rescued
C. got sick because of going for ski outside
D. got rescued immediately the weather turned fine
24. Which of the following statements is TRUE according to the passage?
A. The writer is a travel agent who loves difficult challenges..]
B. In Iceland the weather is always extreme and it snows a lot.
C. Awful journeys may become life-changing events that inspire people.
D. The travelers were so depressed that they needed psychotherapy.

B
In a message beginning “Dear President Obama”, the 86-year-old queen Elizabeth said “I have
been deeply shocked and saddened to learn of the dreadful loss of life today in Newtown.
Connecticut; particularly the news that so many of the dead are children.”
The gunman, named as Adam Lanza, 20, killed 20 small children and six teachers in the Friday
shooting, after walking into a school in an idyllic Connecticut town. He had earlier killed his
mother, who was a teacher at the school.


Former school bus driver Marsha Moskowitz is in disbelief. She once drove at least two of the
victims to school, and remembers driving the gunman when he was in middle school. “I recall
him to be quiet, reserved and very shy. Pretty much stayed to himself on the bus.”
Connecticut State Police spokesman Paul Vance says Lanza forced his way into the building.
“We have established the point of entry. He was not voluntarily let into the school at all,
but he forced his way into the school. But that's as far as we can go on that.”
As the nation struggles to understand how the tragedy could have occurred, Lawrence Palinkas,
Professor of Social Work at the University of Southern California, notes the reason why so many
young Americans have become targets in mass killings is unknown. “Largely because we still don't
have enough data — surprisingly, given the increasing number of events. But generally when
helpless populations are victimized like that,it's usually to send a message, and it's
unfortunately a message of intense anger.” As of now, no one knows the reason why Adam Lanza
did what he did, and many residents of Newtown fear they never will.
The queen, who never gives interviews and normally restricts her public comment to events in
Commonwealth nations, added “Prince Philip joins me in extending our heartfelt sympathy to you
and the American people at this difficult time. The thoughts and prayers of everyone in the United
ingdom and throughout the Commonwealth are with the families and friends of those killed and
with all those who have been affected by today-events. ”
25. What happened to the queen Elizabeth?
A. She was shocked at the crime scene.
B. She could bear the sorrow and hardship.
C. She was determined to fight with crime.
D. She wrote to Obama to show her sympathy.
26. According to the whole text, the gunman __________.
A. was a teacher in a primary school
B. was invited to attend a school lecture
C. appeared more gentle than cruel in the residents' mind
D. once drove two of the victims to school
27. We can learn from the passage that __________.
A. no reason can account for Adam’s killing
B. each American has possessed a gun
C. Adam is always kind to his neighbors
D. such killings seldom happen in the USA


can we conclude from the last paragraph?
A. The queen will attend the victims' funeral ceremony.
B. the queen shows much concern and sympathy to the American people.
C. Maybe it will be illegal for ordinary people to possess guns.
D. More harmless people will be killed during their daily life.

C
A new generation addiction is quickly spreading all over the world. Weboholism, a twentieth century
disease, affects people from different ages. They surf the Net, use email and speak in chat rooms. They
spend many hours on the computer, and it becomes a compulsive habit. They cannot stop, and it affects
their lives.
Decades ago, no one thought that using computers could become compulsive behavior that could affect
the social and physical life of computer users. This obsessional behavior has affected teenagers and college
students. They are likely to log on computers and spend long hours at different websites.
They become hooked on computers and gradually their social and school life is affected by this situation.
They spend all free time surfing and don't concentrate on homework, so this addiction influences their
grades and success at school. Because they can find everything on the websites, they hang out there.
Moreover, this addiction to websites influences their social life.
They spend more time in front of computers than with their friends. The relation with their friends
changes. The virtual life becomes more important than their real life. They have a new language that they
speak in the chat rooms and it causes cultural changes in society.
Because of the change in their behavior, they begin to isolate themselves from the society and live with
their virtual friends. They share their emotions and feelings with friends who they have never met in their
life. Although they feel confident on the computer, they are not confident with real live friends they have
known all their life. It is a problem for the future. This addictive behavior is beginning to affect the entire
world.
29. The main idea of the passage is about __________.
A. the cause of weboholism
B. the advantage of weboholism
C. the influence of weboholism
D. the popularity of weboholism
30. The underlined word
A. addictive B. attractive C. professional D. potential


31. We can infer from the passage that __________.
A. weboholism has the greatest effect on teenagers
B. people are addicted to games on the Internet
C. students can hardly balance real and virtual life
D. virtual life is more vivid and attractive anyway
32. The author's attitude towards weboholism is that of being __________.
A. optimistic


D
News that Microsoft made a $$44.6 billion bid to buy Yahoo resulted in heated discussions made
by many Internet users. Here are some responses
Diane Burke of Weeks bury, entucky
I think it would definitely be an interesting combination. Everyone recognizes the names Yahoo
and Microsoft, but everyone also says, “Did you Google it?” Such a catchphrase (时髦话) is
going to be hard to beat.
Shaun Carney of Laurelville, Ohio
I think the merger (合并) will provide more competitions for Google. I don’t think the merger
will allow Microsoft to develop faster than Google, though. I believe the increased competition
this merger brings will force Google to stay on top of its game by offering more fresh and original
Internet tools and expanding on the tools it already offers.
Toni Suarez of Hacienda Heights, California
I view the merger as a necessary element in preventing a monopolization (垄断). Perhaps it
would bring better high technology innovations to e-mail and help in researching and developing
a better Internet!
Rick R. of Edgewater, Florida
It sounds like a disaster to me. If this were to happen, I would stop using my Yahoo e-mail
account because I don’t like the feeling of Microsoft spying upon my business. I will sign up
for Google.
Antonio Glosser of ansas City, Missouri
Right now, Yahoo offers a lot of features and tools at no cost for all different levels of
Internet users. Microsoft seeks nothing but profit. Undoubtedly, they’ll do nothing other than
B. neutral C. acceptable D. critical


find ways to start removing Yahoo’s formerly non-priced features. Microsoft’s greed will ruin
the great thing that Yahoo currently is.
33. How many companies are mentioned in the passage?
A. Two. B. One. C. Three. D. Four.
34. Which of the following statements is NOT true according to the passage?
A. Toni believes the Internet will have a promising future.[]
B. Shaun’s attitude towards Google is passive.
C. Rick will support Google after the merger.
D. Antonio is afraid that the merger will cost Yahoo’s free features.
35. The passage is mainly about __________.
A. the strengths of Yahoo and Microsoft
B. the strengths and weaknesses of large companies
C. opinions on the merger of Yahoo and Microsoft
D. the future of Yahoo and Microsoft
第三节(共5小题;每小题2分,满分10分)
根据短文内容,从短文后的选项中选出能填入空白处的最佳选项,选项中有两项为多余选项。
Every person has a great deal of love to give. ____36____ Here is a general guide to loving.
Say it. When you say the words “I love you”, make sure you really mean it and are willing to do anything
for that special person.
____37____ If you love someone, you should understand how they feel, where they come from, and who
they are. Realize how they could also love you back just as well.
Love Unconditionally. Giving unconditional love requires you to love without any expectations.
____38____ It seems difficult to only love and not demand anything in return in today’s world where
everyone is selfish. But even in the world we see many examples of unconditional love, for example the love
a mother gives to her child and the love between two life partners. To love unconditionally, you will have to
learn to sacrifice(牺牲) yourself and your desires and also realize that someone may have a different way of
showing his and her love for you. ____39____.
Realize it can be lost. If you realize you can lose the one you love, then you have a greater appreciation of
what you have. ____40____ Don’t make idol of the person you love. This will place them under unnecessary
pressure and will likely result in you losing them. Never stop loving. Even if you have been hurt before. You
should not stop giving love.
A. Try loving just for the sake of love.
B. Put yourself in someone else’s shoes.


C. There are many different ways to explain love .
D. Think how lucky you are to have someone to love.
E. Do not expect to be loved back in exactly the same way.
F. It doesn’t make you a bad person to desire someone else’s love.
G. There are many different ways to love someone(or even yourself).

第二节 完形填空(共20小题;每小题1.5分,满分30分)
阅读下面短文,从短文后各题所给的 四个选项(A、B、C、D)中,选出可以填入空白处的最佳选项,
并在答题卡上将该项涂黑。
In the depths of my memory, many things I did with my father still live. These things have come to
represent, in fact, what I call ___41___ and love.
I don’t remember my father ever getting into a swimming pool. But he did ___42___ the water. Any kind
of ___43___ ride seemed to give him pleasure. ___44___ he loved to fish; sometimes he took me along.
But I never really liked being on the water, the way my father did. I like being ___45___ the water,
moving through it, ___46___ it all around me. I was not a strong ___47___, or one who learned to swim
early, for I had my ___48___. But I loved being in the swimming pool close to my father’s office and
___49___ those summer days with my father, who ___50___ come by on a break. I needed him to see what I
could do. My father would stand there in his suit, the ___51___ person not in swimsuit.
After swimming, I would go ___52___ his office and sit on the wooden chair in front of his big desk,
where he let me ___53___ anything I found in his top desk drawer. Sometimes, if I was left alone at his desk
___54___ he worked in the lab, an assistant or a student might come in and tell me perhaps I shouldn’t be
playing with his___55___. But my father always___56___and said easily, “Oh, no, it is___57___.”
Sometimes he handed me coins and told me to get ___58___an ice cream…
A poet once said, “We look at life once, in childhood; the rest is ___59___.” And I think it is not only
what we “look at once, in childhood” that determines our memories___60___ the person who, in that
childhood, look at us.
41. A. desire
42. A. avoid
43. A. boat
44. A. But
45. A. on














B. joy






C. anger

D. worry
B. refuse
B. bus
C. praise


D. love







D. bike
D. Still
C. train
C. And


B. Then


B. off C. by D. in
D. getting
D. runner
46. A. having
47. A. swimmer
B. leaving

C. making
B. rider C. walker


48. A. hopes
49. A. spending
50. A. should
51. A. next






B. faiths
B. saving



C. rights



D. fears
C. wasting

D. running
B. would



C. had to



D. ought to


D. last B. only
B. out of
C. other
52. A. away from
53. A. put up
C. by

D. inside
D. work out B. break down

C. play with
54. A. the moment B. the first time C. while

D. before
D. fishing net 55. A. lab equipment
56. A. stood up
57. A. fine




B. office things
B. set out
B. strange





C. wooden chair
C. showed up
C. terrible

D. turned out
D. funny
58. A. the student
59. A. memory
60. A. as






B. the assistant
B. wealth

C. myself

D. himself
D. practice
D. And
C. experience
B. but C. or
第二节(共10小题;每小题1.5分,满分15分)
阅读下面材料,在空白处填入适当的内容(不多于3个单词)或括号内单词的正确形式。
Johnny Carson began his career in television ____61____ his twenties. First he worked at local stations in
Nebraska. Several years ____62____, he moved to Los Angeles, California. It was Tonight Show
____63____ made Johnny Carson famous for thirty years. He became ____64____ most popular star of
American television. He ____65____(call)
the country liked him and felt they knew him.
He did not take his fame ____66____( serious). For example, when asked ____67____ he became a
he answered 气体的) state and I cooled.
____68____. Audiences laughed at his jokes he made at the beginning of his show. However, sometimes they
laughed ____69____ harder at the jokes that failed. He was the most powerful performer on television.
Many____70____(comedy) and singers became successful after appearing on

第四部分:写作(共两节 满分35分)
第二节 : 短文改错(共10小题;每小题1分,满分10分)
文中共有10处语言错误,每句中最多有两处。错误涉及一个单词的增加,删除或修改。
增加: 在缺词处加一个漏字符号( ∧ ),并在其下面写出该加的词。


删除:把多余的词用斜线()划掉。
修改: 在错的词下划一横线,并在该词下面写出修改后的词。
注意: 1. 每处错误及其修改均仅限一词;
2. 只允许修改10处,多者(从第11处起)不记分。
This morning, when I am walking on the street I saw that two travelers were reading a map, looking
puzzling. It seemed that they lost. I went up to them and asked how I could help. They told me they were
looking at the Bell Tower. I led them to the nearby bus stop but advised them to take Bus No.20, who could
take them there directly. They appreciated my help too much. Before long, the bus came back. We waved
goodbye to each other. Seen them on a bus, I felt a kind of satisfaction.



第二节: 书面表达(满分25分)
假设你是李华,于搭乘国外某航班(flight number BA793)回国,旅途的糟糕经历令你很不愉快。现
请你用英文给该航空公司写一封投诉信。
信件包括要点如下:
陈述写信原因
投诉内容:
1.航班耽误了7个小时。
2.登机后的不愉快经历。
3.托运的行李箱被损坏。
4.提出你的要求。
参考词汇:check-in luggage

注意:1.词数120 左右
2.可以适当增加细节,使行文连贯.

Dear Sir or Madam,
_______________________ __________________________________________________ __________________
____________________________ __________________________________________________ _____________
_________________________________ __________________________________________________ ________
______________________________________ __________________________________________________ ___
___________________________________________ ________________________________________________


____________________________
________ __________________________________________________ _________________________________
_____________ __________________________________________________ ____________________________
__________________ __________________________________________________ _______________________
___
Yours,
Li Hua



听力:
1-5 ACCCC
阅读理解:
21-24 CBBC 25-28 DCAB 29-32 CACD 33-35 CBC
6-10 ABBCB 11-15 CAABC 16-20 CCBCA
七选五: 36-40 GBAED
阅读理解:
41-45 BDACD 46-50 AADAB 51-55 BDCCB 56-60 CACAB
语法填空:
61. in 62. later 63. that 64. the

65. was called
sly 67. how
短文改错:
68. humor 69. even 70. comedians
71. am –was 72. puzzling—puzzled
75. but—and
80. a—the
76. who—which


73. weregot lost 74. at –for
77. too—very 78. back去掉 79. seen—seeing
书面表达:
Dear Sir or Madam,
My name is Li Hua. I am writing this letter to tell you my terrible flight experience on BA793 on March
3
rd
, 2013.
The first trouble started when the flight was listed as “Delayed”. We were stuck at the airport and no
one came to offer us any explanation. Seven hours later, when all the passengers finally got on board the
plane, We were told we had to keep on waiting for another hour due to the bad weather! If we had been
informed earlier, we would have chosen to stay in the waiting room. What was worse, upon my arrival, I
found my check-in luggage suitcase which was newly bought out of shape, and its handle was broken![]


I am looking forward to an explanation about this unpleasant experience and I strongly suggest your
company do something to provide quality service to the fliers.


高考模拟英语试卷
考试时间100分钟 试题分数 150
卷I
第一部分:听力略
第二部分:阅读理解( 共 2 节,满分40分)
第一节(共15小题;每小题2分,满分30分)
A
Tom goes everywhere with Catherine Green, a 54-year-old secretary. He moves around her office at
work and goes shopping with her.
wonderful.
moves around buying his food, paying his health bills and his taxes, but in fact Tom is a dog.
Catherine and Tom live in Sweden, a country where everyone is expected to lead an orderly life
according to rules laid down by the government, which also provides a high level of care for its people. This
level of care costs money.
People in Sweden pay taxes on everything, so aren't surprised to find that owning a dog means more
taxes. Some people are paying as much as 500 Swedish kronor in taxes a year for the right to keep their dog,
which is spent by the government on dog hospitals and sometimes medical treatment for a dog that falls ill.
However, most such treatment is expensive, so owners often decide to offer health and even life premium
for their dog.
In Sweden dog owners must pay for any damage their dog does. A Swedish ennel Club official explains
what this means if your dog runs out on the road and gets hit by a passing car, you, as the owner, have to
pay for any damage done to the car, even if your dog has been killed in the accident.
21. Catherine pays taxes for Tom because .
A. Tom follows her everywhere B. Tom is her child
C. Tom is her dog D. Tom often falls ill
22. The money paid as dog taxes is used to .
A. keep a high level of care for the people B. pay for damage done by dogs
C. provide medical care for dogs D. buy insurance for dog owners
23. The underlined world .
A. payment for risks B. medical check
C. entertainment expense D. protection fee
24. If a dog causes a car accident and gets killed, who should pay for the damage done to the car?


A. The owner of the car. B. The owner of the dog.
C. The insurance company. D. The government.
B
Parents will do anything for their children. Whether it would be the food, clothes, or the place they
live in, parents will always make sure their children have the best of everything. One of the biggest
concerns that parents currently have is educating their children. They are faced with lots of difficulties
when it comes to education for their young ones. Choices include private school, charter school, public
school or home schooling.
Fortunately for parents,there are a variety of different strategies that they can take in order to
increase their child’s overall intelligence. According to Ross A. Thompson, PhD, professor of psychology at
the University of California at Davis, a child’s brain will actually reach 90% of its full size by the time they
start kindergarten.
Researchers at Northwestern University in Chicago concluded that playing a musical instrument
directly affected the brain stem, the lower section of the brain. Moreover, children that learn an instrument
at an early age tend to continue it when they are older. And numerous studies show that a well balanced
meal is essential for a child’s mental as well as physical health. Some other researchers recommended that
a second language should be taught when children are in preschool. They state that three to four years of
age is the perfect time and that any age under four would work perfectly well for a child’s brain
development.
The average child will learn how to read between kindergarten and second grade. Many parents can
give their children a jump start by teaching them to read before they start school. Children could have an
adequate background of the alphabet and sounds of words before attending school. This can be done by
simply reading to them whenever you want. The more comfortable they get, the easier the process will be.
the first paragraph, we can infer that .




A. parents find it hard to satisfy their kids’ various demands
B. parents meet a new challenge when raising their kids
C. parents have more choices in offering education to their kids
D. parents get actively involved in the process of their kids’ growth
suggestion may the researchers in the passage make?
A. To train kids as early as possible.
B. To try as many means as possible.
C. To help kids develop as fully as possible.
D. To use as many available resources as possible.


is the third paragraph mainly about?




A. The proper time to teach children to learn languages.
B. The effective ways to help develop children’s brains.
C. The vital factors to have a great effect on children’s health.
D. The practical steps to arouse children’s interest in study.
can replace the words “jump start” in the last paragraph with .
A. inspiration B. introduction C. suggestion
C
When I was small and my grandmother died, I couldn't understand why I had no tears. But that
night when my dad tried to cheer me up, my laughs turned into crying.
So it came as no surprise to learn that researchers believe crying and laughing are controlled by the
same part of the brain. Just as laughing has many health advantages, scientists are discovering that so does
crying.
Whatever helps us to reduce pressure is important to our emotional health, and crying seems to work
well. One study found that 85 percent of women and 73 percent of men report feeling better after crying.
Besides, tears attract help from other people. Researchers agree that when we cry, people around us
become kinder and friendlier and they are more ready to provide support and comfort. Tears also enable us
to understand our emotions better; sometimes we don't even know we're very sad until we cry. We learn
about our emotions through crying, and then we can deal with them.
Just as crying can be healthy, not crying ---holding back tears of anger, pain or suffering ---can be bad
for physical health. Studies have shown that too much control of emotions can 1ead to high blood pressure,
heart problems and some other illness. If you have a health problem, doctors will certainly not ask you to
cry. But when you feel like crying, don't fight it. It's a natural, healthy and emotional response.
29. Why didn't the author cry when her grandmother died?
A. Because her father did not want her to feel too sad.
B. Because she did not love her grandmother.
C. Because she was too shy to cry at that time.
D. The author doesn't give the explanation.
can be inferred from the text that .
A. there are two ways to keep healthy
B. emotional health has a close relationship to physical health
C. crying has many health disadvantages.
D. crying does more good to health than laughing
D. promotion


31. According to the author, which of the following statements is true?
A. Crying is the best way to get help from others.
B. Fighting back tears may cause some health problems.
C. We will never know our deep feelings unless we cry.
D. We must cry if we want to reduce pressure
32. What might be the most suitable title for the text?
A. Why We Cry B. How to eep Healthy
C. Power of Tears D. New Scientific Discovery
D
As early as the mid-18th century, some people began raising doubts about Marco Polo’s travels.
In1995,historian Frances Wood argued in her book
Did Marco Polo Go to China ?
that the famous explorer from
Venice never made it to pass the Black Sea. She noted that his travel journal left out the Great Wall of
China, chopsticks and tea drinking among other details. Furthermore, Chinese documents from Polo’s day
made no mention of the explorer and his men.
Wood and other scholars have argued that Marco Polo based his tales of China on information
collected from fellow trades who had actually been there. Last year, a team of Italian researchers became
the latest to challenge Polo’s accounts(叙述). They said that evidence didn’t support his description of ublai
han’s Japanese invasions (
侵略
).
Now, however, research by Hans Ulrich Vogel of Germany’s Tubingen University might help prove
Marco Polo was true. In a new book
Marco Polo Was in China
,the professor of Chinese history tries
to prove that Marco Polo spoke the truth. He suggests, for example, that Polo didn’t include the Great Wall
in his book because it only achieved its great importance in the Ming Dynasty several hundred years later.
Vogel further explains that Chinese records from the 13th and 14th centuries avoided setting down visits
from Westerners.
Historians before him have touched on these issues. But Vogel also relies on another evidencethe
explorer’s very detailed descriptions of currency and salt production in the Yuan Dynasty. According to
Vogel, Polo documented these aspects of Mongol Chinese culture in greater detail than any other of his
time. This is a hint (暗示) that Polo relied on his own powers of observation.
Will we ever know whether Marco Polo traveled to China? Perhaps not, but the consequences of his
real or fictional journey are still felt across the globe. One reader of The Travels of Marco Polo was
Christopher Columbus, who stepped upon the New World while following his idol

s footsteps.
Wood doubted Marco Polo’s travel’s to China because his description .
A. missed some important culture of China


B. covered so much about traders’ life
C. was full of obvious mistakes
D. seemed less detailed
’s trust on Marco Polo is based on the argument that .
a. The Great Wall didn’t gain its importance then
b. Records in the Yuan Dynasty mentioned Polo
c. Polo mentioned the currency and salt
d. Polo’s other works are believable
e. Polo recorded what he saw in great detail
A. a, b, d B. a, c, d C. a, e D. b, c


of the following shows the structure of the text? (P1为第一段,以此类推)
A B C D
第二节(共
5小题;每小题
2分,满分10分)
In high school, it’s important to stay healthy. 36 How can you study well if you’re sick? So you
should pay attention to your health in order to study well in high school. In order to enjoy good health, you
should have some good habits. Here are some tips for you.
Drink water regularly every day. Some students don’t like drinking water. They only drink water
when they are extremely thirsty. 37 You should drink water regularly so that your body and brain can
function well. Avoid sports drinks and soda when you are thirsty. They are not good for your health.
38 Some students don’t eat three meals a day. No matter what happens, you should not skip any
meal of the day. Eating regularly helps keep your metabolism (新陈代谢) high and keeps your energy up.
It’s especially important to eat a good breakfast every day with protein, carbohydrates (碳水化合物), and
fiber to set up your body for the day of activity. If you don’t eat, how can you stay focused in class?
Eat the right foods. Some students eat whatever they can get. This is wrong. 39 Choose whole
grains like rice and whole wheat flours. Skip muffins, donuts, and other processed foods. Instead, choose
whole grains, protein, fruits and vegetables.
40 If you do that, you can get distracted. Therefore, you may not pay attention to how much you are
eating. You may eat too much. Eating too much harms your health. And you may gain too much weight
because of it. Try to eat slowly so your brain sends the right signals to tell your belly that it’s full.


A. Don’t skip meals.
B. This is bad for your health.
C. Eat a good breakfast every day.
D. Avoid eating too much at one time.
E. Don’t watch TV or movies when you eat.
F. If you are unhealthy, you may fall ill easily.
G. You should eat the foods that are good for your body.
第一节:完形填空(共20小题;每小题1.5分,满分30分)
You had better learn to be grateful. If ___41____, you naturally open yourself up to receive all kinds
of blessings and good things in life. You can receive almost everything you want__42__. If you want
recovery soon, start by feeling grateful that you are still__43__. If it is more money that you want, start
being grateful for whatever __44__of money you already have.
You see, showing gratitude(感恩) is the key to developing __45__attitude to life. It is the key to
building successful relationships, and it is the key to __46__healthier and happier lives. And this is __47__
the importance of being grateful lies. In a word, it makes life better.
One of my favorite__48__is “If you learn to appreciate more of what you already have, you will find
yourself having more to appreciate.” Think about that!
Something as__49__as a “Thank you for being in my life” or “I__50__having you here with me”, can
go a long way. It will make the other person feel good, and__51__it also will make you feel better!
So maybe your wife is not very__52__around the house, but she’s great at cooking your favorite__53__.
Or, maybe your child spilled milk all over your new shirt right __54____ you walked out of the door for
work, but usually he makes you laugh so much and makes you__55__the luckiest parent in the world.
I strongly__56__you to make a promise to yourself for the next week. Decide to__57__every morning
and think of at least 5 things that you are__58__for. Then say __59___out loud. You will notice a smile on
your face without even trying. In 30 days you will feel completely renewed. This is the__60__of being
grateful!
41. A. so
42. A. correctly
43. A. lovely
44. A. amount
45. A. terrible
46. A. feeling
47. A. what
48. A. mottos
49. A. pleasant
50. A. average
51. A. for sure
52. A. lazy
53. A. milk
B. any
B. truly
B. live
B. plenty
B. confusing
B. looking
B. where
B. word
B. delighted
B. avoid
B. regardless of
B. organized
B. drinks
C. possible
C. wholly
C. lively
C. kind
C. positive
C. enjoying
C. which
C. habits
C. simple
C. appreciate
C. instead of
C. careless
C. bread

D. necessary
D. clearly
D. alive
D. number
D. negative
D. taking
D. that
D. rules
D. wonderful
D. admit
D. by way of
D. beautiful
D. dishes


54. A. if
55. A. become of
56. A. persuade
57. A. dress up
58. A. grateful
59. A. one
60. A. disadvantage

B. after
B. look like
B. suggest
B. wake up
B. upset
B. it
B. consequences
C. before
C. act as
C. demand
C. get up
C. anxious
C. them
C. award
D. unless
D. feel like
D. advise
D. make up
D. concerned
D. that
D. power
第二节 语法填空在空白 处填入适当的内容
(1
个单词)或括号内单词的正确形式。请把答案写在答题纸上!!
(满分15)

I am fond __61__my sister but she has one serious shortcoming. She can be really stubborn. I still
remember what happened last month. Although she didn’t know __62__ best way of getting to some places,
she insisted on __63__(organize) a trip. I kept asking her when we were leaving , __64__ we were coming
back, and whether she had looked at a map. To my questions, she gave no answer. Instead, she gave me a
__65__(determine) look---which said she would not change __66__mind. When I told her that our journey
would begin at an altitude of more than 5,ooo __67___(meter), she seemed to be excited. When I told her
the air would be hard __68__(breathe) and it would be very cold, she said it would be a __69__(delight)
experience. I know my sister __70__(good) than anyone else. Once she has made up her mind, nothing can
change it.

第三部分:写作(共两节,满分35分) 请在答题纸上作答!!
第一节:短文改错(共10小题;每小题1分,满分10分)
短文中 共有10处错误,每句中最多有两处。错误涉及一个单词的增加、删除或修改。请按照下列格式
修改:
增加:在缺词处加一个漏词符号(∧),并在其下面写出该加的词。
删除:把多余的词用斜线()划掉。
修改:在错的词下画一横线,并在该词下面写出修改后的词。
注意:每处错误及其修改均仅限一词; 只允许修改10处,多者(从第11处起)不计分。
I was playing at my cousin house. Since his family was rich than mine, he had more toys than I did.
There was one on particular I’d always wanted. I put into my pocket when he wasn’t looking. I guessed,
even at that age, I would never be able to enjoy to playing with the toy or faced my cousin again; I would
always know I’d done something wrong. Late on, my aunt drove me home. When she dropped me off, I
pulled out the toy slow and gave it back. She knows what had happened, but she thanked me and ever
mentioned it again.



第二节:书面表达(满分25分)
假定你是小林,你的朋友李华马上要参加高考了 ,因此他很紧张,晚上经常睡不着觉。现在,请你给
他发一封邮件,教他如何克服压力(get over pressure),内容包括:
1.对李华表示安慰和鼓励
2.介绍消除压力的方法
3.提一些生活和学习方面的建议
4.提出希望
注意:不得写出真实班级和姓名;可以适当增加情节,使行文连贯;词数100 左右,给出的开头和结尾不
计入总词数。

Dear Li Hua,
I am sorry to learn that_____________________________________
____ _____________________________________________ __________
___________________ < br>_______________________________________________ ______________________________
_______________ __________________________________ __________
___________________ < br>_______________________________________________ ______________________________

Best wishes,
Yours,
iao Lin

语法填空

61.____________ 62 _______________
63. ___________ 64. ______________
65. ___________ 66._______________
67.____________ 68 _______________
69.____________ 70. ______________


短文改错:(满分10分)

I was playing at my cousin house. Since his family was rich than mine, he had more toys than I did.
There was one on particular I’d always wanted. I put into my pocket when he wasn’t looking. I guessed,
even at that age, I would never be able to enjoy to playing with the toy or faced my cousin again; I would
always know I’d done something wrong. Late on, my aunt drove me home. When she dropped me off, I
pulled out the toy slow and gave it back. She knows what had happened, but she thanked me and ever
mentioned it again.









书面表达:(满分25分)

Dear Li Hua,
I am sorry to learn that_____________________________________
____ _____________________________________________ __________
___________________ < br>_______________________________________________ _______________________________
______________ ___________________________________ __________
___________________
__________________________________________________ ____________________________
_________________ ________________________________ __________
___________________
__________________________________________________ ____________________________
_________________ ________________________________ __________
___________________



Best wishes,
Yours,
iao Lin

听力略.
阅读 CCAB BABD DBBC ACB FBAGE
完形ABDAC CBACC ABDCD DBACD
语篇填空 of the organizing when determined
her meters to breathe delightful better
改错1. cousin →cousin’s 2. rich →richer 3. on →in 4. put ∧into →itthat 5. enjoy to playing
→去掉to 6. faced →face 7. Late →Later 8. slow →slowly 9. knows →knew
→never
作文
Dear Li Hua,
I am sorry to learn that you are feeling so nervous that you can’t sleep well at night as the
College Entrance Examination draws near. But in fact, you don’t have to worry about it, and it
is not as hard as you think.
In order to get over the pressure, you should live a regular life and take more exercise to
keep relaxed. What’s more, you had better have a balanced diet, and milk, meat, vegetables
as well as fruit are all good choices. Time permitting , you can chat with your parents or friends
about how you feel
Believe in yourself and make full preparations! I am sure that you will succeed .
Best wishes
iao Lin


高考模拟英语试卷
审题人:唐竹生(市教科院)
时量:120分钟 分值:150分
注意:本试卷分为四个部分,包括听力、语言知识、 阅读理解和书面表达。答题前,考生须将答题卷
密封线内的各项内容填写清楚,所有答案按要求正确清晰 地填写在答题卷上。考试结束后,将本试卷和答
题卷一并交回。

Part I Listening Comprehension (30 marks)
Section A (22.5 marks)
Directions In this section, you will hear six conversations between two speakers. For each conversation,
there are several questions and each question is followed by three choices marked A, B and C. Listen
carefully and then choose the best answer for each question.
You will hear each conversation TWICE.
Example
When will the magazine probably arrive?
A. Wednesday.
The answer is B.
Conversation 1
1. What has the weather been like these days?
A. Windy. . C. Rainy.
B. Thursday. C. Friday.
2. What will the temperature be tomorrow?
A. 8℃.
Conversation 2
3. Who is visiting the woman for the weekend?
A. Her teacher. B. Her student. C. Her brother.
B. 10℃. C. 12℃.
4. How does the woman know Ann?
A. She has just been introduced to her.
B. She has taken piano lessons from her.
C. They have met at a party before.
Conversation 3
5. When did the man finish running every day last term?
A. At 700. B. At 730. C. At 810.


6. What does the woman think of his university life?
A. Too dull.
Conversation 4
7. Where are the two speakers?
A. In a library. B. In a bookstore. C. In an office.
B. Just so so. C. Very great.
8. Why didn’t the woman lend him the three books?



A. Because they are expensive.
B. Because they are very important.
C. Because they are single copies.
9. How many books did the man finally take away?
A. One. B. Three. C. Four.
Conversation 5
10. What’s the relationship between the two speakers?



A. Ticket seller and passenger.
B. Saleswoman and customer.
C. Close friends.
11. Which flight is the man going to take?
A. The early morning flight. B. The mid- morning flight. C. The early afternoon flight.
12. Where does the conversation most probably take place?
A. At the bus station. B. At the ticket office. C. At the city centre.
Conversation 6
13. What was the woman’s first job?
A. Sales assistant. B. Sales manager.
14. Why did the woman give up the job?
A. To travel with her friends.
B. To prepare for her exams.
C. To attend a meeting.
15. Where did the woman stay?
A. In England. B. In America. C. In Ireland.

Section B (7.5 marks)
Directions In this section, you will hear a short passage. Listen carefully and then fill in the numbered
blanks with the information you have heard. Fill in each blank with NO MORE THAN THREE WORDS.
C. Sales representative.


You will hear the short passage TWICE.
Newspapers in Btitain
● Serious newspapers on happenings both 16 and abroad.
Types
● 17 newspapers on entertainment.
● Beginning in 18 .
● Enjoying a good reputation for believable news and serious opinions.
The Times ● Not giving its 19 to a political party.
● Talking about new fashions and 20 of the young people
occasionally.


Part II Language nowledge (45 marks)
Section A (15 marks)
Directions Beneath each of the following sentences there are four choices marked A,B, C and D.
Choose the one answer that best completes the sentence.
Example
The wild flowers looked like a soft orange blanket the desert.
A. covering
The answer is A.
21. Tom took a taxi to the high-speed train station, only his train had just left.
A. finding B. to find C. found D. find
B. covered C. cover D. to cover
22. Food supplies in the tsunami-stricken area . We must act immediately before there’s none left.
A. have run out B. run out C. ran out D. are running out
23. Though to see us, the professor gave us a warm welcome.
A. surprising B. surprised C. to surprise D. being surprised
24. Tom has always been working hard. , he will hopefully achieve good grades in the coming test.
A. However B. Otherwise C. Therefore D. Besides
25. Lily in a top school in Changsha, but she is studying in our school now.
A. studies B. will study C. has studied D. studied
26. Mo Yan had been awarded the Nobel Prize for Literature made us Chinese proud.
A. What B. Which C. That D. Whether
27. People will keep fit if they to eat more fruit and vegetables.
A. persuade B. will persuade


C. will be persuaded D. are persuaded
28. I was feeling left out in the new school Alice, an easygoing girl from Canada, came to stay with
me.
A. as B. once C. when D. while
29. We lost our way in the forest park, otherwise we more places of interest yesterday.
A. visited B. had visited C. would visit D. would have visited
30. Only after we have gone through hard times something to be grateful for.
A. will we understand
C. we understood




B. we will understand
D. did we understand
31. Most riders of electric bikes in China 40-50 kilometers per hour often ignore red lights.
A. reaching B. reached C. to reach D. to have reached
32. difficulties we may meet with in our life, we should keep a positive attitude towards life.
A. However B. Whatever C. Whichever D. Whenever
33. — Mum, I really think Dad should have a break and get relaxed.
— Yes. He too long.
A. has been reading
C. had read






B. read
D. is reading
34. The driver from that bus company, was extremely calm and brave in face of danger, saved all
the passengers on board.
A. which B. who C. whom D. that
35. Joe, along with his friends bound to find nobody there, for the rest gone for their
lunch.
A. is; have

Section B (18 marks)
Directions For each blank in the following passage there are four words or phrases marked A, B, C
and D. Fill in each blank with the word or phrase that best fits the context.
It was a warm sunny Saturday afternoon about fifteen years ago. I offered to take my 36
to the local playground. As soon as we got there, my daughter headed for the 37 and asked for a push.
As I was helping my daughter to 38 higher and higher, I noticed another little girl trying in vain to
get her own swing(秋千) going. Her elderly grandmother was sitting quietly on a nearby bench and smiled
at me.
I walked over to the little girl and 39 if she could use a push, too. She smiled and answered “Yes!”
B. is; has C. are; have D. are; has

陷阱英文-我亲爱的英文


vehicle是什么意思-dgi


goth-dpm是什么


gypsophila什么意思-文静是什么意思


persuad-enemies什么意思啊


lizard怎么读-感谢的近义词


辫子的意思-high


anaL什么意思-西班牙语歌曲



本文更新与2020-11-02 16:22,由作者提供,不代表本网站立场,转载请注明出处:https://www.bjmy2z.cn/gaokao/436360.html

【附20套高考模拟试题】贵州省2020年7月普通高中学业水平考试英语试卷含答案的相关文章

  • 爱心与尊严的高中作文题库

    1.关于爱心和尊严的作文八百字 我们不必怀疑富翁的捐助,毕竟普施爱心,善莫大焉,它是一 种美;我们也不必指责苛求受捐者的冷漠的拒绝,因为人总是有尊 严的,这也是一种美。

    小学作文
  • 爱心与尊严高中作文题库

    1.关于爱心和尊严的作文八百字 我们不必怀疑富翁的捐助,毕竟普施爱心,善莫大焉,它是一 种美;我们也不必指责苛求受捐者的冷漠的拒绝,因为人总是有尊 严的,这也是一种美。

    小学作文
  • 爱心与尊重的作文题库

    1.作文关爱与尊重议论文 如果说没有爱就没有教育的话,那么离开了尊重同样也谈不上教育。 因为每一位孩子都渴望得到他人的尊重,尤其是教师的尊重。可是在现实生活中,不时会有

    小学作文
  • 爱心责任100字作文题库

    1.有关爱心,坚持,责任的作文题库各三个 一则150字左右 (要事例) “胜不骄,败不馁”这句话我常听外婆说起。 这句名言的意思是说胜利了抄不骄傲,失败了不气馁。我真正体会到它

    小学作文
  • 爱心责任心的作文题库

    1.有关爱心,坚持,责任的作文题库各三个 一则150字左右 (要事例) “胜不骄,败不馁”这句话我常听外婆说起。 这句名言的意思是说胜利了抄不骄傲,失败了不气馁。我真正体会到它

    小学作文
  • 爱心责任作文题库

    1.有关爱心,坚持,责任的作文题库各三个 一则150字左右 (要事例) “胜不骄,败不馁”这句话我常听外婆说起。 这句名言的意思是说胜利了抄不骄傲,失败了不气馁。我真正体会到它

    小学作文
【附20套高考模拟试题】贵州省2020年7月普通高中学业水平考试英语试卷含答案随机文章